Vous êtes sur la page 1sur 59

Morphopathology - CS

Tamir Bessisy 1246

1. Which of the following cellular changes is considered to be pre-cancerous?


a. [ ] Anaplasis
b. [X] Dysplasia
c. [ ] Metaplasia
d. [ ] Hyperplasia
e. [ ] Hypertrophy
2. Which of the following is true about renal cell carcinoma?
a. [ ] The kidney is commonly compressed crescentically at one pole of the tumor.
b. [ ] It most commonly arises in adult males with a peak in the 6th decade of life
c. [ ] The clear cell type is among the most common types
d. [ ] The sarcomatoid type is among the rarest types
e. [X] All of the above
3. Which of the following is true about renal cell carcinoma?
a. [ ] It most commonly arises near the renal hilum
b. [ ] The peak incidence of the tumor is the third decade of life
c. [ ] Renal cell carcinoma of the right may be associated with the right varicocele
d. [ ] Collecting duct type carcinoma is the most frequent type
e. [X] None of the above
4. Which of the following diseases is characterized by proteinuria?
a. [X] Multiple myeloma
b. [ ] Liver cirrhosis
c. [ ] Urinary bladder stone
d. [ ] All of the above
e. [ ] None of the above
5. Which of the following tumors may consist of cells with pale or clear cytoplasm?
a. [ ] Seminoma
b. [ ] Renal cell carcinoma
c. [ ] Krukenbergs tumor
d. [X] All of the above
e. [ ] None of the above
6. Which of the following statements is true about endometrial hyperplasia?
a. [ ] It is the most common in females using contraceptive pills
b. [ ] The type known as cystic hyperplasia is strongly precancerous
c. [ ] The condition predisposes to endometriosis
d. [ ] All of the above
e. [X] None of the above
7. Which of the following ovarian tumors can be bilateral?
a. [ ] Mucinous cystadenoma
b. [ ] Krukenberg tumor
c. [ ] Ovarian carcinoma
d. [X] All of the above
e. [ ] None of the above
8. Which of the following tumors may show a cartilaginous component?
a. [ ] Nephroblastoma
b. [ ] Hepoblastoma
c. [ ] Osteosarcoma
d. [ ] Teratoma
e. [X] All of the above

1
Morphopathology - CS
Tamir Bessisy 1246

9. Which of the following characterized by considerable lymphocytic infiltrates?


a. [ ] Grave’s disease
b. [ ] Hashimoto’s disease
c. [ ] Medullary carcinoma of the breast
d. [X] All of the above
e. [ ] None of the above
10. Which of the following disease may be complicated by lymphoma?
a. [ ] Riedel’s thyroiditis
b. [X] Hashimoto’s thyroiditis
c. [ ] De Quervein’s thyroiditis
d. [ ] All of the above
e. [ ] None of the above
11. Which of the following is true about papillary carcinoma of thyroid?
a. [ ] The tumor is more aggressive than follicular carcinoma of thyroid
b. [ ] The tumor cells have eosinophilic cytoplasm and dark bizarre nuclei
c. [ ] Lung and bone metastases are common and occur early in the course of disease
d. [ ] All of the above
e. [X] None of the above
12. Which of the following is true about Grave’s disease:
a. [ ] The thyroid gland is mostly asymmetrically enlarged
b. [X] The condition is associated with peritibial myxoedema in 5% of cases
c. [ ] The thyroid follicles are hugely enlarged due to large amounts of retained colloid
d. [ ] Outer and cut surfaces of the gland are nodular
e. [ ] All of the above.
13. Which of the following viruses is related to carcinoma of cervix and tongue?
a. [ ] Ebstein-Barr virus
b. [ ] Cytomegalovirus
c. [X] Human papilloma virus
d. [ ] Herpes virus
e. [ ] All of the above
14. Which is the most following microscopic type of cervical cancer?
a. [ ] Adenocarcinoma
b. [X] Squamous cell carcinoma
c. [ ] Undifferentiated cell carcinoma
d. [ ] Adenocanthoma
e. [ ] Carcinoma in situ
15. Precancerous thyroid disease include:
a. [ ] Papilloma
b. [ ] Polyp
c. [ ] Serous cystadenoma
d. [ ] Folliculitis
e. [X] Adenoma
16. Seminoma is also called:
a. [ ] Apudoma
b. [ ] Leidigoma
c. [X] Disgerminoma
d. [ ] Follicular adenoma
e. [ ] Adenoma

2
Morphopathology - CS
Tamir Bessisy 1246

17. Thyroid cancer usually develops on the background:


a. [ ] Polyp
b. [ ] Papilloma
c. [ ] Basalioma
d. [ ] Insuloma
e. [X] Adenoma
18. Which of the following tumors doesn’t contain stroma?
a. [ ] Hydatiforme mole
b. [ ] Malignant insuloma
c. [X] Choryonepithelioma
d. [ ] Choryoidpapilloma
e. [ ] Medullar carcinoma
19. Serous membranes can be a source of development:
a. [ ] Basalioma
b. [X] Mesothelioma
c. [ ] Meningioma
d. [ ] Osteosarcoma
e. [ ] Synovioma
20. Choose immature tumor of mesenchymal origin:
a. [ ] Fibroma
b. [X] Sarcoma
c. [ ] Lipoma
d. [ ] Leiomyoma
e. [ ] Carcinoma
21. The term cancer means
a. [ ] Cell division
b. [ ] Cell multiplication
c. [ ] Out of control
d. [X] Crab
e. [ ] Lobster
22. A malignant epithelial cell neoplasm derived from any of the three germ layers is referred to as:
a. [ ] Sarcoma
b. [ ] Carcinoma
c. [X] Teratoma
d. [ ] Mixed cell tumor
e. [ ] Adenoma
23. The study of neoplastic growths is referred to as:
a. [ ] Tetralogy
b. [ ] Anaplasia
c. [X] Oncology
d. [ ] Neoplasia
e. [ ] Dysplasia
24. A benign epithelial cell neoplasm derived from non-glandular surfaces is referred to as:
a. [X] Papilloma
b. [ ] Sarcoma
c. [ ] Adenoma
d. [ ] Hamartoma
e. [ ] Squamous cell carcinoma

3
Morphopathology - CS
Tamir Bessisy 1246

25. Which one of the following is not considered to be a distinctive pattern of non-neoplastic growth?
a. [ ] Regeneration
b. [ ] Hypertrophy
c. [ ] Hyperplasia
d. [X] Anaplasia
e. [ ] Metaplasia
26. Each of the following is an anaplastic change except:
a. [ ] Pleomorphism and hyperchromatism
b. [ ] Increased mitosis and abnormal mitotic figures
c. [ ] Nuclei that vary in shape and size
d. [ ] Presence of undifferentiated cells
e. [X] Presence of abundant chromatin in cytoplasmic organelles
27. Liver nodules that consist of congenital localized overgrowth of mature hepatocytes are referred to as:
a. [ ] Hepatomas
b. [X] Hamartomas
c. [ ] Benign sarcomas
d. [ ] Nodular hyperplasia
e. [ ] Bile duct anaplasia
28. Which of the following is least likely to be used as a means of distinguishing a benign from a
malignant neoplasm?
a. [ ] Degree of cellular differentiation
b. [ ] Rate of growth
c. [X] Type and amount of necrosis
d. [ ] Evidence of metastasis
e. [ ] Mode of spread
29. Which one of the following neoplasms is highly invasive but seldom spread by metastisis?
a. [ ] Papilloma of the skin
b. [ ] Squamous cell carcinomas of the skin
c. [ ] Adenocarcinomas of the lungs
d. [X] Basal cell carcinomas of the skin
e. [ ] Osteogenic sarcomas of the limbs
30. Which one of the following features is more characteristic of a benign than a malignant neoplasm?
a. [ ] Grows by expansion and implantation occurs frequently
b. [ ] Metastasizes if the brain is the site of origin
c. [ ] Usually non-encapsulated and necrosis seldom occurs
d. [ ] Tend to recur after surgical removal
e. [X] Usually occur singly and do not recur after surgical removal
31. Which one of the following is considered to be the hallmark of malignancy?
a. [ ] Anaplasia and the rate of growth of the neoplastic mass
b. [ ] Metastasis and the degree of encapsulation of the neoplastic mass
c. [ ] Formation of giant cells and cellular anaplasia within and around the neoplastic mass
d. [X] Presence of undifferentiated cells and evidence of metastasis
e. [ ] Cellular anaplasia and growth by expansion of the neoplastic mass
32. The process of neoplastic cells moving through the circulatory system and becoming lodged in a vessel
causing obstruction is referred to as:
a. [ ] Anaplasia
b. [ ] Neoplasia
c. [ ] Thrombosis
d. [ ] Transplantation
e. [X] Embolism

4
Morphopathology - CS
Tamir Bessisy 1246

33. The process by which glandular epithelium of the prostate is transformed into squamous epithelium
following prolonged administration of estrogens is known as:
a. [ ] Neoplasia
b. [ ] Dysplasia
c. [ ] Hyperplasia
d. [X] Metaplasia
e. [ ] Anaplasia
34. A neoplasm characterized by the presence of "cancer pearls" and intercellular bridges is most likely a:
a. [ ] Basal cell epithelioma
b. [ ] Pheochromocytoma
c. [X] Squamous cell carcinoma
d. [ ] Histocytoma
e. [ ] Perianal gland adenoma
35. What does pleomorphism mean?
a. [ ] Uncontrolled mitosis
b. [ ] Multiple nuclei
c. [X] Variability in shape and size
d. [ ] The cells are different from where they arose from
e. [ ] All of the above
36. What is the process called by which cells move from one site to another?
a. [ ] Transportation
b. [ ] Biotransformation
c. [X] Metastasis
d. [ ] Metrostatic
e. [ ] Flagellum
37. What is a metastatic adenocarcinoma of the stomach that specifically goes to the ovary called?
a. [ ] Metastatic adenocarcinoma
b. [ ] Melanoma
c. [X] Krukenberg tumor
d. [ ] Wilson's tumor
e. [ ] Richtsler's tumor
38. If a tumor is benign and of squamous origin, what would it be called?
a. [ ] Malignant
b. [ ] Krukenburg
c. [ ] Adenoma
d. [X] Papilloma
e. [ ] Carcinoma
39. If a tumor is benign and glandular in origin, what is it called?
a. [ ] Malignant
b. [ ] Krukenburg
c. [X] Adenoma
d. [ ] Carcinoma
e. [ ] Papilloma
40. Benign tumor of cartilage is called an:
a. [ ] Osteoma
b. [X] Chondroma
c. [ ] Leiomyoma
d. [ ] Lipoma
e. [ ] Melanoma

5
Morphopathology - CS
Tamir Bessisy 1246

41. Benign tumor in smooth muscle is called a:


a. [ ] Osteoma
b. [X] Leiomyoma
c. [ ] Lipoma
d. [ ] Chondroma
e. [ ] Rabdomyoma
42. What is it called when the nucleus are pushed off to one side due to abundant mucin?
a. [ ] Hypertrophy
b. [ ] Benign
c. [ ] Krukenburgs sign
d. [X] Signet-rings
e. [ ] Wilsons-circles
43. What does TNM stand for?
a. [ ] Tumor size, lymph node, malignancy
b. [ ] Tumor size, leimyoma, malignancy
c. [ ] Tumor shape, lymph node, metastasis
d. [X] Tumor size, lymph node, metastasis
e. [ ] This is not a medical grading system
44. The following are true about atherosclerosis:
a. [ ] It occurs in the retinal artery
b. [ ] Foamy macrophages are seen in type i plaque
c. [ ] Thinning of the intima is a feature
d. [X] Proliferation of smooth muscle cells in the intima is typical
e. [ ] Raised hdl is associated with atherosclerosis
45. The following are true about cerebral infarction:
a. [X] The area of infarct tends to be wedge-shaped
b. [ ] It can result from thrombosis of the external carotid artery
c. [ ] disease
d. [ ] Coagulative necrosis occurs in the brain tissue
e. [ ] Cortical blindness can result from infarction of the anterior cerebral artery
46. The primary anatomic site of pressure regulation in the vascular system is:
a. [ ] Aorta
b. [ ] Arteries
c. [X] Arterioles
d. [ ] Capillaries
e. [ ] Heart
47. Complications of chronic hypertension include each of the following except:
a. [ ] Left ventricular hypertrophy
b. [ ] Congestive heart failure
c. [ ] Renal failure
d. [X] Diabetes mellitus
e. [ ] Brain hemorrhage
48. Causes of secondary hypertension include all of the following except:
a. [ ] renal artery stenosis
b. [ ] adrenal cortical carcinoma
c. [ ] chronic pyelonephritis
d. [X] lipoid nephrosis (“nil” disease)
e. [ ] chronic glomerulonephritis

6
Morphopathology - CS
Tamir Bessisy 1246

49. Patchy destruction of elastic tissue in the aortic media associated with obliterative endarteritis of
vaso vasorum is characteristic of:
a. [ ] Marfan’s syndrome
b. [ ] Giant cell arteritis
c. [X] Syphilitic aortitis
d. [ ] Polyarteritis nodosa
e. [ ] Monckeberg’s sclerosis
50. Which one of the following is most often the cause of death in cases of dissecting aneurysms?
a. [X] Hemopericardium
b. [ ] Congestive heart failure
c. [ ] Myocardial infarction
d. [ ] Aortic stenosis
e. [ ] Aortic coarctation
51. The most common cause of abdominal aortic aneurysms is:
a. [ ] Trauma
b. [X] Atherosclerosis
c. [ ] Syphilis
d. [ ] Hypertension
e. [ ] Cystic medial necrosis
52. Aneurysms of the arch of the aorta are most characteristically caused by
a. [ ] Atherosclerosis
b. [ ] Tuberculosis
c. [X] Syphilis
d. [ ] Congenital defects
e. [ ] Fungi
53. Which of the following does not appear to be a risk factor in the development and complications of
atherosclerosis?
a. [ ] Maleness
b. [ ] Diabetes mellitus
c. [ ] Hypertension
d. [X] Alcoholism
e. [ ] Cigarette smoking
54. Following injury produced by cutting the toe nail too short, a 70-year-old diabetic woman developed infection
that progressed to gangrene of the left great toe. The most likely vascular disease process is:
a. [ ] Infectious arteritis
b. [ ] Thrombophlebitis
c. [X] Arteriosclerosis
d. [ ] Phlebosclerosis
e. [ ] Thromboangitis obliterans
55. The major cause of pulmonary thromboemboli is:
a. [ ] Hypertension
b. [ ] Heart failure
c. [ ] Atherosclerosis
d. [X] Thrombophlebitis
e. [ ] Varicose veins
56. Severe (malignant) hypertension is characterized by:
a. [X] Hyperplastic arteriolosclerosis
b. [ ] Aortic insufficiency
c. [ ] Marfan’s syndrome
d. [ ] Calcific aortic stenosis

7
Morphopathology - CS
Tamir Bessisy 1246

e. [ ] Thromboangiitis obliterans
57. Acute coronary occlusion is followed by:
a. [ ] myocardial gangrene
b. [X] myocardial infarction
c. [ ] brown atrophy
d. [ ] myocardial hypertrophy
e. [ ] heart lipomatosis
58. Which atherosclerotic stage can be complicated by aneurysm?
a. [ ] Lipidic
b. [ ] Fibrolipidic
c. [ ] Atheromatous
d. [X] Ulcerative
e. [ ] Necrotic
59. Coronary artery thrombosis is followed by:
a. [ ] Gangrene
b. [X] Infarction
c. [ ] Hemosiderosis
d. [ ] Lipomatosis
e. [ ] Brown atrophy
60. Designate pathological processes that may develop in the myocardium due to hypertension:
a. [X] Myocardial infarction
b. [ ] Gangrene
c. [ ] Hemorrhage
d. [ ] Atrophy
e. [ ] Vicarious hypertrophy
61. Specify cardiosclerosis types:
a. [ ] Postinfarction
b. [ ] Macrofocal
c. [ ] Vicarious
d. [ ] Microfocal
e. [X] All of the above
62. Which of the following processes is characteristic for arterial hypertension?
a. [ ] Thrombophlebitis
b. [ ] Phlebothrombosis
c. [X] Elastofibrosis
d. [ ] Atherocalcinosis
e. [ ] All of the above
63. Which myocardial infarction stage may be complicated by rupture of heart wall?
a. [ ] Allergic
b. [ ] Functional
c. [X] Necrotic
d. [ ] Organization
e. [ ] Ossification
64. Which type of rheumatic fever does chorea refer to?
a. [ ] Joint
b. [ ] Cardiovascular
c. [ ] Arthritic
d. [ ] Nodose
e. [X] Cerebral

8
Morphopathology - CS
Tamir Bessisy 1246

65. Depending on the prevalence of tissue reactions rheumatic endocarditis can be:
a. [ ] Hemorrhagic
b. [ ] Purulent
c. [ ] Fibrinous
d. [X] Serous
e. [ ] Putrid
66. Which of the following organ is always affected in rheumatic fever?
a. [ ] Kidney
b. [ ] Skin
c. [X] Heart
d. [ ] Brain
e. [ ] Lung
67. What are the characteristic morphological changes in the kidney in systemic lupus erythematous?
a. [ ] Fibroplastic nephritis
b. [ ] Lupus nephritis
c. [X] Lipoid nephritis
d. [ ] Extracapillary glomerulonephritis
e. [ ] Pyelonephritis
68. Which of these changes characterize decompensated heart disease:
a. [ ] Extension of the heart cavities
b. [ ] Dropsy of the cavities
c. [ ] Lipidic dystrophy of the myocardium
d. [ ] Eccentric hypertrophy
e. [X] All of the above
69. The following statements are true about poststreptococcal glomerulonephritis except
a. [ ] It is an autoimmune complex disease
b. [X] Acute renal failure is the common fate
c. [ ] The glomeruli appear microscopically hypercellular
d. [ ] It mostly affects children and young adult
e. [ ] Oliguria and hematuria are features of the disease
70. Which of the following statements is true about chronic glomerulonephritis?
a. [ ] The disease affects both kidneys, often in an asymmetrical pattern
b. [ ] Chronic glomerulonephritis represents the most common fate of post streptococcal glomerulonephritis
c. [X] Clinically, the patient may develop hypertension and may complain of polyuria
d. [ ] All the above
e. [ ] None of the above
80. All of the following are features of renal failure except:
a. [ ] Alkalosis
b. [ ] High blood urea
c. [ ] Anemia
d. [ ] Inflammation of serous membrane
e. [X] Gastrointestinal inflammation
81. Choose the most common complication in GN
a. [ ] Pneumonia
b. [X] Renal failure
c. [ ] Pyelonephritis
d. [ ] Adrenal failure
e. [ ] Liver failure

9
Morphopathology - CS
Tamir Bessisy 1246

82. The most common causes of necrotic nephrosis is the following EXCEPT:
a. [ ] Poisons
b. [X] Xerophtalmia
c. [ ] Severe infections
d. [ ] Massive hemolysis
e. [ ] Traumatic lesions
83. Which of the following about pyelonephritis is right?
a. [ ] Infectious pathology with glomerules involvement
b. [ ] Immune mesangial disease
c. [X] Pathology of the renal pelvis, calices and interstitial tissue
d. [ ] Pathology of the convolute tube
e. [ ] Lesions of the renal pelvis mucosal layer
84. The macroscopic changes of the kidneys in acute pyelonephritis are the following EXCEPT:
a. [ ] Increased in size
b. [ ] Hyperemia of the parenchyma
c. [X] Renal pelvis dilation
d. [ ] Renal pelvis contains mucus
e. [ ] Mycroabscesses on cut section
85. The microscopic changes of the kidneys in acute pyelonephritis are the followings EXCEPT:
a. [ ] Hyperemia of the mucosal layer
b. [ ] Leucocyts infiltration of the mucosa
c. [ ] Epithelial degeneration
d. [X] Nonspecific granulomatosis
e. [ ] Necrosis of the mucosal layer
86. The macroscopic changes of the kidneys in chronic pyelonephritis are the following EXCEPT:
a. [ ] The kidneys are different in size
b. [ ] The surface is macronodular
c. [ ] Renal pelvis is enlarged
d. [ ] Thickness of the renal pelvis wall
e. [X] The capsule can be easy removed
87. The microscopic changes of the kidney in chronic pyelonephritis are the following EXCEPT:
a. [ ] Sclerosis of the renal pelvis mucosa
b. [X] Pytuitary-like appearance of the kidney
c. [ ] Thyroid-like
d. [ ] Vassals sclerosis
e. [ ] Distrophiy and atrophy of the tube
88. The complications in acute pyelonephritis are the following EXCEPT:
a. [ ] Renal abscesses
b. [ ] Pyonephrosis
c. [ ] Perinephritis
d. [X] Hepatitis
e. [ ] Sepsis
89. The morphologic changes in nephrolithiasis include the following EXCEPT:
a. [X] Renal parenchyma hypertrophy
b. [ ] Renal pelvis dilatation
c. [ ] Hydronephrosis
d. [ ] Renal parenchyma atrophy
e. [ ] Pyelonephritis

10
Morphopathology - CS
Tamir Bessisy 1246

90. The complications in nephrolitiasis are the following EXCEPT:


a. [ ] Pyelonephritis
b. [X] Glomerulonephritis
c. [ ] Pyonephrosis
d. [ ] Sepsis
e. [ ] Chronic renal failure
91. The glomerulopathy include:
a. [X] Glomerulonephritis
b. [ ] Myelomatous kidney
c. [ ] Tubule fermentopathy
d. [ ] Polycystic kidney disease
e. [ ] Kidney litiasis
92. "Thyroid-like" kidney is caused by:
a. [ ] Renal amyloidosis
b. [ ] Basedow goiter
c. [X] Chronic pyelonephritis
d. [ ] Acute pyelonephritis
e. [ ] Extracapillary glomerulonephritis
93. What is the acute kidney insufficiency most severe complication?
a. [ ] Amyloidosis
b. [ ] Primary contracted kidney
c. [ ] Nephrolithiasis
d. [X] Total cortical necrosis of kidney
e. [ ] Kidney carbuncle
94. Chronic glomerulonephritis is characterized by:
a. [ ] Macronudal kidney surface
b. [ ] Primary shrunken kidneys
c. [X] Secondary shrunken kidneys
d. [ ] Large tallow kidney
e. [ ] Large spotted kidney
95. Which of the following changes are found in uremia?
a. [ ] Hemorrhagic diathesis
b. [ ] Fibrinous pericarditis
c. [ ] Fibrinous pneumonia
d. [ ] Pulmonary edema
e. [X] All of the above
96. All of the following causes hematuria EXCEPT:
a. [ ] Urinary stones
b. [ ] Cystitis
c. [X] Nephrotic syndrome
d. [ ] Urinary neoplasm
e. [ ] Leukemia
97. Which of the following statement is true about crescent glomerulonephritis?
a. [ ] It is characterized by glomerular necrosis
b. [ ] The bowman's capsule show parietal crescent
c. [ ] The disease often progresses rapidly to renal failure
d. [X] All of the above
e. [ ] None of the above

11
Morphopathology - CS
Tamir Bessisy 1246

98. Opportunistic infections occur when:


a. [ ] Pathogen infect the host
b. [X] Resident flora cause infectious disease
c. [ ] Bacteria are spread by poor hand washing
d. [ ] Transient bacteria spread among hopitalized patients
e. [ ] Bacteria affect persons in condition of low temperature
99. An epithelialized track connecting the midportion of the jejunum and a point on the skin three cm left
of the umbilicus is an example of a(n):
a. [ ] autolysis
b. [ ] choristoma
c. [ ] cyst
d. [X] fistula
e. [ ] supernumerary structure
100. The "acute phase reaction" in acute inflammation is a group of biochemical changes mediated by:
a. [ ] dilatation of small blood vessels
b. [X] factors released from macrophages
c. [ ] histamine and complement components, among others
d. [ ] neutrophil injury to tissue
e. [ ] the increased erythrocyte sedimentation rate
101. What's the characteristic cell in tissue inflammation caused by worms?
a. [X] eosinophil
b. [ ] lymphocyte
c. [ ] macrophage
d. [ ] neutrophil
e. [ ] plasma cell
102. Which of the following is associated with acute inflammation?
a. [X] Neutophils
b. [ ] Macrophages
c. [ ] Lymphocytes
d. [ ] Tissue fibrosis
e. [ ] Tissue necrosis
103. Acute inflammation may be triggered by infections, trauma, physical or chemical agents, tissue necrosis,
foreign bodies, and immune reactions. Which of the following is NOT seen in acute inflammation?
a. [ ] Alterations in vascular caliber
b. [X] Decrease in blood flow
c. [ ] Structural changes in the microvasculature (edema)
d. [ ] Plasma proteins and leukocytes leaving the circulation
e. [ ] Leukocyte focusing to eliminate the offending agent
104. Vascular changes associated with acute inflammation include ____ (from histamine and NO) and ____
vascular permeability.
a. [ ] Vasoconstriction; Decreased
b. [ ] Vasoconstriction; Increased
c. [ ] Vasodilation; Decreased
d. [X] Vasodilation; Increased
e. [ ] Vasodilatation; Normal
105. Which of the following is NOT a general principle of the chemical mediators of inflammation?
a. [ ] Mediators originate either from plasma or from cells
b. [ ] The production of active mediators is triggered by microbial products or by host proteins
c. [ ] One mediator can stimulate the release of other mediators by target cells themselves
d. [ ] Mediators can act on one or few target cell types

12
Morphopathology - CS
Tamir Bessisy 1246

e. [X] Once activated and released from the cell, most of these mediators last a long time (long-lived)
106. Which of the following is NOT true regarding contribution to inflammation?
a. [ ] Lysosomal constituents increase vascular permeability and tissue damage
b. [ ] Oxygen free radicals amplify the cascade that elicits the inflammatory response
c. [ ] Neuropeptides help initiate and propagate the inflammatory response
d. [X] The response to hypoxia decreases vascular permeability
e. [ ] The response to necrotic cells is pro-inflammatory
107. Which of the following appears, histologically, as an eosinophilic meshwork of threads or sometimes as an
amorphous coagulum?
a. [ ] Serous inflammation
b. [X] Fibrinous inflammation
c. [ ] Suppurative or purulent inflammation
d. [ ] Ulcers
e. [ ] Gangrenous inflammation
108. Which of the following is characterized by the production of large amounts of pus consisting of
neutrophils, necrotic cells, and edema fluid?
a. [ ] Serous inflammation
b. [ ] Fibrinous inflammation
c. [X] Suppurative or purulent inflammation
d. [ ] Ulcers
e. [ ] Gangrenous inflammation
109. Which of the following is marked by the outpouring of a thin fluid that, depending on the size of injury, is
derived from either the plasma or the secretions of mesothelial cells lining the peritoneal, pleural, and
pericardial cavities?
a. [X] Serous inflammation
b. [ ] Fibrinous inflammation
c. [ ] Suppurative or purulent inflammation
d. [ ] Ulcers
e. [ ] Gangrenous inflammation
110. Which of the following is NOT a systemic effect of inflammation?
[ ] Fever
[ ] Increased acute-phase proteins
[ ] Leukocytosis
[X] Decreased pulse and blood pressure
[ ] Cytokine release (IL-1 and TNF)
111. Which of the following would be associated with defective inflammation?
[X] Infections
[ ] Allergies
[ ] Asthma
[ ] Psoriasis
[ ] Vitiligo
112. At necropsy, a central focal area of coagulative necrosis of the myocardium surrounded by proliferating
fibroblast and angioblast indicates that the infarction occurred:
a. [ ] 1 to 2 hours before death
b. [ ] 7 to 24 hours before death
c. [X] 7 to 14 days before death
d. [ ] 25 to 48 hours before death
e. [ ] more than 5 months before death

13
Morphopathology - CS
Tamir Bessisy 1246

113. The pain usually associated with an inflammatory reaction can best be explained by:
a. [ ] Damage To Nerve Endings In Direct Contact With The Inflammatory Agent
b. [X] Combined effect of increased tissue pressure and certain chemical mediators (e.g. bradykinin)
c. [ ] Liberation Of Serotonin From Mast Cell
d. [ ] Direct Action of lysosomal enzymes
e. [ ] Direct Action Of Histamine And Fragments Of Complement
114. During the formation of a scar, the contraction of granulation tissue is accompanied by:
a. [ ] proliferation of capillaries
b. [X] contractile proteins within fibroblasts
c. [ ] collagen fibers
d. [ ] fibrin and collagen fibers
e. [ ] epithelioid cells and giant cells
115. Development of tensil strength in a healing wound parallels the:
a. [X] deposition of collagen fibers
b. [ ] deposition of mucopolysaccharides
c. [ ] growth and proliferation of fibroblast and angioblast
d. [ ] deposition of reticulum fibers
e. [ ] contraction of the scar
116. Which of the following is not considered to be a cardinal sign of inflammation?
a. [ ] pain
b. [ ] heat
c. [X] cold
d. [ ] reddness
e. [ ] swelling
117. The cardinal signs of inflammation are most likely to be associated with
a. [X] acute inflammatory reactions
b. [ ] chronic inflammatory reactions
c. [ ] granulomatous inflammatory reactions
d. [ ] healed wound
e. [ ] subacute inflammatory reactions
118. The cell type that occurs with the least degree of frequency in a chronic inflammatory reaction is:
a. [ ] fibroblasts
b. [ ] angioblasts
c. [ ] macrophages
d. [ ] lymphocytes
e. [X] neutrophils
119. Epithelioid cells are derived from:
a. [ ] microglial cells
b. [X] blood monocytes
c. [ ] t-lymphocyte
d. [ ] activated basophiles
e. [ ] giant cells
120. The inflammatory cell type considered to be the "hallmark" of acute inflammation is:
a. [ ] epithelioid cell
b. [ ] myofibroblasts
c. [X] neutrophils
d. [ ] b-lymphocytes
e. [ ] eosinophils

14
Morphopathology - CS
Tamir Bessisy 1246

121. Which of the following has the capacity to reproduce at the site of injury?
a. [ ] neutrophiles and macrophages
b. [ ] lymphocytes and eosinophiles
c. [X] basophiles and neutrophils
d. [ ] macrophages and lymphocytes
e. [ ] plasma cells and macrophages
122. Which one of the following is active in both intrinsic blood coagulation enzyme cascade and the kallikrein system?
a. [ ] Thrombin
b. [ ] antihemophilic globulin (factor viii)
c. [ ] stuart-prower factor (factor x)
d. [X] hageman factor (factor xii)
e. [ ] platelet factor - 3
123. The characteristic feature of early hemodynamic changes in acute inflammation is:
a. [ ] decreased flow of blood to the affected tissue
b. [ ] increased permeability of capillaries and venules
c. [ ] release of histamine into the affected tissue
d. [ ] exudation of neutrophils
e. [X] increased blood flow to the affected tissue
124. In the earlier stages of acute inflammation, histamine is responsible for increased permeability in:
a. [ ] veins and capillaries
b. [X] venules and capillaries
c. [ ] large arteries and arterioles
d. [ ] arterioles and venules
e. [ ] veins and large arteries
125. Which one of the following cell types is not an active component of inflammatory process:
a. [ ] neutrophils
b. [ ] basophiles
c. [X] erythrocytes
d. [ ] monocytes
e. [ ] eosinophiles
126. Which one of the following cell types responds to chemotactic influences by converting proesterase-i to
serine esterase:
a. [ ] macrophages
b. [X] neutrophils
c. [ ] basophiles
d. [ ] monocytes
e. [ ] eosinophiles
127. A factor released from igE sensitized basophiles or mast cells serves as a chemotactic force for:
a. [ ] monocytes
b. [X] eosinophiles
c. [ ] plasma cells
d. [ ] t-lymphocytes
e. [ ] neutrophils
128. Which one of the following is least likely to function as an efficient chemotactic agent?
a. [ ] products derived from bacteria
b. [ ] products of the complement system
c. [ ] a factor present in lysates of neutrophils
d. [ ] factors liberated by sensitized lymphocytes
e. [X] bradykinin derived from alpha 2-globulin

15
Morphopathology - CS
Tamir Bessisy 1246

129. The process by which mobile leukocytes escape from the blood vessel lumen into perivascular tissue is
appropriately referred to as:
a. [X] emigration
b. [ ] migration
c. [ ] margination
d. [ ] emploperisis
e. [ ] chemotaxis
130. The engulfment of foreign particulate matter by inflammatory cells is appropriately referred to as:
a. [ ] emploperisis
b. [X] phagocytosis
c. [ ] chemotaxis
d. [ ] leukocytosis
e. [ ] anaplasia
131. Each of the following is true for a fibrinous exudate except:
a. [ ] fibrin is the major constituent
b. [ ] associated with severe inflammatory reactions
c. [X] major constituent of a phlegmon
d. [ ] occurs primarily on mucous and serous membranes
e. [ ] Aids in localizing bacteria and serves as a framework for repair processes
132. Each of the following is a feature of suppurative exudation except:
a. [ ] presence of neutrophiles
b. [ ] presence of pyogenic bacteria
c. [ ] death of cells
d. [ ] liquefaction
e. [X] diphtheric membranes
133. An exudate characterized by excessive production of mucin is appropriately referred to as:
a. [X] catarrhal exudate
b. [ ] serous exudate
c. [ ] non-inflammatory exudate
d. [ ] hemorrhagic exudate
e. [ ] purulent exudate
134. Which of the following has the greatest capacity to regenerate:
a. [ ] renal glomeruli
b. [ ] myocytes of heart
c. [ ] skeletal muscle cells
d. [ ] neuron of the central nervous system
e. [X] hepatocytes
135. Note the appearance of exudate underlying diphtheritic inflammation:
a. [ ] Purulent
b. [ ] Serous
c. [X] Fibrinous
d. [ ] Hemorrhagic
e. [ ] Katharal
136. Flegmon is characterized by:
a. [ ] Khataral inflammation
b. [ ] Fibrinous inflammation
c. [ ] Fibro- connective delimitation
d. [ ] Pyogenic membrane presence
e. [X] Presence of diffuse purulent inflammation

16
Morphopathology - CS
Tamir Bessisy 1246

137. Which of the following is the initial inflammatory step?


a. [ ] Exudation
b. [ ] Proliferation
c. [X] Alteration
d. [ ] Phagocytosis
e. [ ] Pinocytosis
138. Specify exudative inflammation type that is typical for throat diphtheria:
a. [ ] Purulent
b. [ ] Khataral
c. [ ] Crupous
d. [X] Diphtheric
e. [ ] Putrid
139. Secondary syphilis is characterized by:
a. [ ] Solitary gummas
b. [ ] Chancre
c. [ ] Eosinophil’s infiltration
d. [ ] Milliary gummas
e. [X] Syphilides
140. Echinococcosis primarily affects:
a. [ ] Lungs
b. [ ] Kidneys
c. [X] Liver
d. [ ] Stomach
e. [ ] Eyes
141. Which organs are most commonly affected by visceral syphilis
a. [ ] Respiratory system
b. [ ] Gastro-ntestinal system
c. [X] Cardiovascular system
d. [ ] Urogenital system
e. [ ] Endocrin system
142. „Cuirass” pericardium is characterised for:
a. [ ] Fibrinous inflammation
b. [X] Organization and calcification of exudate
c. [ ] Suppuration
d. [ ] Adderence formation
e. [ ] Tumor spreading
143. The most frequent causes of purulent inflammation are:
a. [ ] Viruses
b. [ ] Toxins
c. [ ] Protozoa
d. [ ] Chemicals
e. [X] Staphylococi
144. Microscopy purulent exudate is diagnosed by a large number of:
a. [ ] Fibrin
b. [ ] Lymphocytes
c. [X] Neutrophyls
d. [ ] Eritrocytes
e. [ ] Thrombocytes

17
Morphopathology - CS
Tamir Bessisy 1246

145. The most frequent causes of purulent inflammation are the following EXCEPT:
a. [ ] Staphylococi
b. [X] Viruses
c. [ ] Escherichia
d. [ ] Streptococci
e. [ ] Klebsiella
146. Cellulitis is referred to the following type of inflammation:
a. [ ] Catarrhal
b. [X] Purulent
c. [ ] Crupous
d. [ ] Gangrenous
e. [ ] Diphtheric
147. Precancerous change of epithelium in chronic catarrh is:
a. [ ] Atrophy
b. [X] Dysplasia
c. [ ] Dystrophy
d. [ ] Desquamation
e. [ ] All of the above
148. Exudate containing a large amount of neutrophilic leukocytes is called:
a. [ ] Serous
b. [ ] Hemorrhagic
c. [X] Purulent
d. [ ] Fibrinous
e. [ ] Gangrenous
149. Which of the followings is most commonly associated with an early complete hydatidiform mole?
a. [ ] Intravillous cistern formation and scalloping of villous surface
b. [ ] Polarization of trophoblastic column
c. [ ] Grape-like vesicles on gross examination
d. [X] Mild circumferential trophoblastic hyperplasia on the villous surface
e. [ ] An in situ choriocarcinoma
150. Which one of the following endometrial lesions is associated with the highest risk of developing
endometrial carcinoma?
a. [ ] Chronic endometritis
b. [X] Complex hyperplasia with atypia
c. [ ] Complex hyperplasia without atypia
d. [ ] Simple hyperplasia
e. [ ] Squamous metaplasia
151. Female and male genital tract inflammatory disease include the following EXCEPT:
a. [ ] Endometritis
b. [ ] Orchitis
c. [X] Cystitis
d. [ ] Prostatitis
e. [ ] Salpingitis
152. Morphologic pattern in chronic endometritis includes the following EXCEPT:
a. [ ] Inflammatory infiltration
b. [ ] Epithelial desquamation and proliferation
c. [ ] Glands atrophy
d. [X] Significant hemorrhage
e. [ ] Cysts formation

18
Morphopathology - CS
Tamir Bessisy 1246

153. Breast Carcinoma lymphogenic metastases appear in the following lymph node EXCEPT:
a. [ ] Fossa axillary lymph node
b. [ ] Anterior chest lymph node
c. [X] Occipital lymph node
d. [ ] Subclavicular lymph node
e. [ ] Parasternal lymph node
154. Which statement concerning breast cancer is correct?
a. [ ] High grade tumors are well differentiated
b. [ ] Poorly differentiated tumors have positive lymph nodes when they are diagnosed
c. [X] Low grade tumors have better prognosis
d. [ ] For grading of breast carcinoma, the following information are used: nuclear grade, tubule
formation and nodal status
e. [ ] All of the above
155. Which of the following statements is true about the partial type of vesicular mole:
a. [ ] The chromosomal pattern is often diploid (46XX or 46XY)
b. [ ] Fetus is often absent
c. [ ] The condition is commonly transformed into invasive mole
d. [ ] All chorionic villi are transformed into vesicles
e. [X] None of the above
156. Which of the following conditions is true about mammary sclerosing adenosis:
a. [ ] This sclerotic mammary lesion follows suppurative mastitis
b. [ ] It may be clinically and microscopically mistaken for fibroadenoma
c. [X] It can be differentiated from invasive cancer by the presence of myoepithelial cells
d. [ ] All of the above
e. [ ] None of the above
157. Which of the following benign mammary neoplasms is usually huge-sized?
a. [ ] Intracanalicular fibroadenoma
b. [ ] Pericanalicular fibroadenoma
c. [X] Phylloides tumor
d. [ ] Dark papilloma
e. [ ] None of the above
158. Which of the following mammary neoplasms has the highest incidence of bilaterality?
a. [X] Lobular carcinoma
b. [ ] Medullary carcinoma
c. [ ] Paget’s disease
d. [ ] Mucoid carcinoma
e. [ ] Apocrine carcinoma
159. Which of the following types of mammary carcinoma may have a good prognosis?
a. [ ] Juvenile secretory carcinoma
b. [ ] Medullary carcinoma
c. [ ] Tubular or mucoid carcinoma occurring in a pure form (not mixed with other type)
d. [X] All of the above
e. [ ] None of the above
160. Which of the following conditions does not indicate poor prognosis of breast carcinoma?
a. [ ] Inflammatory carcinoma
b. [ ] Skin invasion
c. [X] Microcalcifications
d. [ ] Cancer en cuirasse
e. [ ] Nipple retraction

19
Morphopathology - CS
Tamir Bessisy 1246

161. Which of the following is not a risk factor for mammary carcinoma?
a. [X] Multiparity
b. [ ] Obesity
c. [ ] Late menopause
d. [ ] Positive family history
e. [ ] Early menarche
162. Which of the following sites shows the highest incidence of breast cancer involvement?
a. [ ] Lower outer quadrant of the breast
b. [X] Upper outer quadrant of the breast
c. [ ] Lower inner quadrant of the breast
d. [ ] Upper inner quadrant of the breast
e. [ ] Central compartment of the breast
163. Which of the following mammary lesions is most liable to be mistaken histologically for carcinoma?
a. [ ] Phylloides tumor
b. [X] Sclerosing adenosis
c. [ ] Gynecomastia
d. [ ] Fibrocystic disease
e. [ ] Mammary duct ectasia
164. Which of the following statement is true about cervical carcinoma?
a. [ ] The endophytic types have better prognosis than the exophytic types
b. [ ] The risk of cancer development is higher in virgins than non-virgins
c. [X] The patients may die from renal failure
d. [ ] All of the above
e. [ ] None of the above
165. Which of the following statement is true about cervical erosion?
a. [X] This is superficial ulceration representing an early manifestation of cervical cancer
b. [ ] This is cervical inflammation associated with epithelial damage and bleeding
c. [ ] This is cervical ulceration as result of puerperal sepsis
d. [ ] This is eversion of cervix as a result of inflammation and fibrosis
e. [ ] None of the above
166. Ectopic pregnancy causes may be:
a. [ ] Inflammatory disease of the fallopian tubes
b. [ ] Scarring of the fallopian tube bends
c. [ ] Developmental abnormalities of the fallopian tubes
d. [ ] Tumor of the fallopian tubes
e. [X] All of the above
167. Horyonepithelioma may develop as a result of:
a. [X] Hydatidiform mole
b. [ ] Endometritis
c. [ ] True cervical erosion
d. [ ] Pseudo- erosion
e. [ ] Adenomatous polyp
168. The most common brain tumor in adults is:
a. [X] Low grade astrocytoma
b. [ ] Meningioma
c. [ ] Metastatic carcinoma
d. [ ] Glioblastoma muliforme
e. [ ] Oligodendroglioma

20
Morphopathology - CS
Tamir Bessisy 1246

169. The following tumor(s) occurs commonly in the ventricles and in the hilum terminale of the spinal cord:
a. [ ] Pilocytic astrocytoma
b. [ ] Hemangioblastoma
c. [ ] Oligodendroglioma
d. [ ] Meduloblastoma
e. [X] Ependimoma
170. A brain tumor which sometimes contain psammoma bodies is (are):
a. [ ] Hemangioblastoma
b. [X] Meningioma
c. [ ] Germ cell tumor
d. [ ] Primary brain lymphoma
e. [ ] Pineoblastoma
171. The following list of primary malignancies accounts for the majority of metastatic brain tumors:
a. [X] Lung, breast, melanoma
b. [ ] Tests, ovary, melanoma
c. [ ] Lung, prostate, uterus
d. [ ] Pancreas, melanoma, uterus
e. [ ] Salivary glands, ovary, testis
172. What is the most common primary intracranial tumor in adults?
a. [X] Meningioma
b. [ ] Ependimoma
c. [ ] Pineoblastoma
d. [ ] Craniopharingioma
e. [ ] Glioblastoma multiforme
173. The following tumor(s) contain perivascular pseudorosettes:
a. [X] Ependimoma
b. [ ] Oligodendroglioma
c. [ ] Glioblastoma multiforme
d. [ ] Meningioma
e. [ ] Pilocitic astrocitoma
174. The following statements is/are true regarding oligodendroglioma:
a. [ ] Account for less than 15% of gliomas
b. [ ] Usually a slow growing tumor, has better prognosis than astrocytoma
c. [ ] May be circumscribed and usually contain calcification
d. [ ] Has a predilection for white matter and histologically tumor cells have a "fried egg" appearance due
to surrounding by a clear cytoplasmic halo
e. [X] All of the above
175. Which of the following histologic features of hepatocellular injury is prognostically least favourable?
a. [ ] Councilman body formation
b. [ ] Bile infarct formation
c. [X] Collagen formation
d. [ ] Ballooning degeneration of hepatocytes
e. [ ] Lobular inflammatory cell infiltrates
176. Which of the following the hepato-renal syndrome refers to?
a. [X] Functional failure of a morphology normal kidney associated with severe liver disease
b. [ ] Simultaneous toxic damage to the liver and kidneys with functional failure of both
c. [ ] Immune complex glomerulopathy from chronic antigenemia associated with chronic viral hepatitis
d. [ ] Acute tubular necrosis due to hypotension after a gastrointestinal blled in a cirrhotic patient
e. [ ] All of these

21
Morphopathology - CS
Tamir Bessisy 1246

177. All of the following statements about fulminant viral hepatitis are true EXCEPT:
a. [ ] It is more common than fulminant hepatitis caused by drugs
b. [ ] Its severity is proportional to the immune response to the virus
c. [X] Death usually within 24 hours of the onset of symptoms
d. [ ] Histologically, it is commonly indistinguishable from drug induced fulminant hepatitis
e. [ ] Survivors usually have lifelong immunity to recurrent infection
178. Causes of cirrhosis in infancy include all of the following EXCEPT:
a. [X] Wilson's disease
b. [ ] Alpha antitritrypsin deficiency
c. [ ] Total parenteral nutrition
d. [ ] Extrahepatic biliary atresia
e. [ ] Galactosemia
179. Which of the following conditions is Malloy's hyaline found in within hepatocytes?
a. [ ] Carbon tetrachloride toxicity
b. [ ] Wilson's disease
c. [ ] Viral hepatitis
d. [X] Alcoholic liver disease
e. [ ] Liver cirrhosis
180. Which of the following types of liver tumors is most commonly associated with the oral contraceptives?
a. [ ] Bile duct adenoma
b. [ ] Bile duct hamartroma
c. [ ] Focal nodular hyperplasia
d. [ ] Hepatocellular carcinoma
e. [X] Hepatoccelular adenoma
181. The most common tumor of the liver is:
a. [ ] Cholangiocarcinoma
b. [ ] Hepatocellular carcinoma
c. [ ] Hemangiosarcoma
d. [ ] Liver cell adenoma
e. [X] Metastatic carcinoma
182. In patients with the most common form of gallstones, the liver secretes more:
a. [ ] Bile salts
b. [ ] 7 alpha - hydroxylase
c. [ ] Low density lipoproteins
d. [X] Cholesterol
e. [ ] Calcium carbonate
183. Alcoholic fatty liver is associated with each of the following EXCEPT:
a. [X] decreased membrane phospholipids formation
b. [ ] decreased fatty acid oxidation
c. [ ] increased triglyceride synthesis
d. [ ] decreased lipoprotein synthesis
e. [ ] increased mobilization of fatty acids from periphery
184. The hepatorenal syndrome is associated principally with:
a. [ ] Microvesicular fatty liver
b. [ ] Intrahepatic cholestasis
c. [ ] Hepatocellular carcinoma
d. [X] Cirrhosis
e. [ ] Extrahepatic biliary obstruction

22
Morphopathology - CS
Tamir Bessisy 1246

185. Unconjugated bilirubin is derived principally from:


a. [ ] Glucuronyl transferase activity
b. [ ] Toxic liver injury
c. [X] Breakdown of senescent red blood cells
d. [ ] Decreased intracellular bilirubin transport
e. [ ] Decreased ligandin
186. Which of the following is associated with destructive cholangitis
a. [ ] Hepatitis B
b. [ ] Alcoholic hepatitis
c. [X] Primary biliary cirrhosis
d. [ ] Neonatal hepatitis
e. [ ] Dubin-Johnson syndrome
187. Central sclerosis of the liver diseases is associated with
a. [ ] Hepatitis B
b. [X] Alcoholic hepatitis
c. [ ] Autoimmune hepatitis
d. [ ] Neonatal hepatitis
e. [ ] Dubin-Johnson syndrome
188. The most common cause of hepatocellular carcinoma
a. [X] Hepatitis B
b. [ ] Alcoholic hepatitis
c. [ ] Autoimmune hepatitis
d. [ ] Neonatal hepatitis
e. [ ] Dubin-Johnson syndrome
189. Mallory's hyaline is associated with:
a. [ ] Autoimmune hepatitis
b. [X] Alcholic hepatitis
c. [ ] Hepatitis B
d. [ ] Hepatitis D
e. [ ] Non-A, non-B hepatitis (Hepatitis C)
190. Predominantly unconjugated hyperbilirubinemia is typical of:
a. [ ] Intravascular hemolysis
b. [ ] Carcinoma of common bile ducts
c. [ ] Carcinoma of gall bladder
d. [ ] Carcinoma of the head of the pancreas
e. [X] Viral hepatitis
191. Ballooned hepatocytes and acidophilic bodies found in a liver biopsy are most indicative of:
a. [ ] Alcoholic hepatitis
b. [X] Acute viral hepatitis
c. [ ] Primary biliary cirrhosis
d. [ ] Hemochromatosis
e. [ ] Cardiac cirrhosis
192. The liver of a patient with right-sided heart failure shows:
a. [ ] Acute necrosis
b. [X] Centrilobular congestion
c. [ ] Portal vein thrombosis
d. [ ] Chronic inflammation in the periportal zone
e. [ ] Nodular regeneration

23
Morphopathology - CS
Tamir Bessisy 1246

193. So-called "bile infarcts" are associated with which of the following?
a. [ ] Drug injury
b. [ ] Hepatitis B
c. [ ] Alcoholic hepatitis
d. [X] Extrahepatic biliary obstruction
e. [ ] Wilson's disease
194. Hepatitis A is transmitted primarily by which of the following routes?
a. [ ] Blood transfusions
b. [ ] Snake bites
c. [X] Fecal-oral
d. [ ] Sexual transmission
e. [ ] Intravenous drug abuse
195. Which of the following regarding acidophilic bodies is true?
a. [ ] They consist primarily of Mallory's hyaline
b. [X] They are necrotic hepatocytes
c. [ ] They occur primarily in the setting of chronic persistent hepatitis
d. [ ] They can be seen in 25-50% of normal livers
e. [ ] They are formed by cytokeratin
196. Histologic features of acute alcoholic hepatitis include each of the following EXCEPT:
a. [ ] Fatty change
b. [ ] Mallory bodies
c. [ ] Infiltration by neutrophils
d. [X] Infiltration by monocytes
e. [ ] Injured hepatocytes
197. Extrahepatic biliary obstruction is caused by each of the following EXCEPT:
a. [ ] Pancreatic carcinoma
b. [ ] Carcinoma of the ampulla of Vater
c. [ ] Bile duct carcinoma
d. [X] Advanced cirrhosis
e. [ ] Sclerosing cholangitis
198. Which of the following conditions could lead to the development of portal hypertension?
a. [ ] Cirrhosis
b. [ ] Portal vein thrombosis
c. [ ] Severe right sided heart failure
d. [ ] Hepatic vein thrombosis (Budd-Chiari syndrome)
e. [X] All of the above
199. You are evaluating a liver biopsy from a patient with acute viral hepatitis. Which of the following would
you expect to see?
a. [ ] Abscesses
b. [ ] Granulomas
c. [X] Lymphocytes in portal tracts
d. [ ] Regenerative nodules
e. [ ] Well-developed scar tissue
200. You are caring for an elderly gentleman with a history of ischemic heart disease who has an enlarged and
tender liver. You notice that he also has edema of the lower extremities. Gross examination of his liver
would most likely reveal:
a. [ ] Fibrosis
b. [ ] Inflammatory exudates
c. [ ] Nodules
d. [X] "Nutmeg" pattern

24
Morphopathology - CS
Tamir Bessisy 1246

e. [ ] Tumor
201. You are examining a patient with advanced cirrhosis. What would you expect to find?
a. [ ] Cervical lymphadenopathy
b. [X] Distended abdomen with fluid wave
c. [ ] Massive hepatomegaly
d. [ ] Muscular hypertrophy
e. [ ] Small spleen
202. It is likely that a gallstone will produce jaundice if impacted in any of the following anatomic sites, EXCEPT:
a. [ ] Ampulla of Vater
b. [ ] Common bile duct
c. [ ] Common hepatic duct
d. [ ] Confluence of common bile duct and pancreatic duct
e. [X] Cystic duct
203. In a patient with a history of viral hepatitis, why is it important to know which virus was involved?
a. [ ] To determine the probability of progressive chronic hepatitis
b. [ ] To have a better idea of therapeutic options
c. [ ] To determine whether the patient can transmit the disease to others
d. [ ] To assess risk of malignancy
e. [X] All of the above
204. Which of the following is LEAST likely to be associated with portal hypertension due to liver cirrhosis?
a. [ ] Ascites
b. [X] Pulmonary hypertension
c. [ ] Spontaneous bacterial peritonitis
d. [ ] Thrombocytopenia
e. [ ] Hepatoprenal syndrome
205. Which of the following is associated with the highest rate of progression to chronic hepatitis?
a. [ ] Hepatitis A virus
b. [ ] Hepatitis B virus
c. [X] Hepatitis C virus
d. [ ] Hepatitis D virus
e. [ ] Hepatitis G virus
206. Hepatic steatosis is characterized by:
a. [ ] Proteic dystrophy of hepatocytes
b. [X] Lipidic dystrophy of hepatocytes
c. [ ] Mineral dystrophy of hepatocytes
d. [ ] Liver hemosiderosis
e. [ ] Glycogen infiltration of hepatocytes
207. Which of the following is true regarding asthma?
a. [ ] It produces dysplastic changes in the respiratory epithelium
b. [ ] It is a frequent cause of bronchiectasis
c. [X] It may be triggered by infection
d. [ ] It causes centrilobular emphysema
e. [ ] All of the above
208. Which of the following is commonly associated with panacinar emphysema?
a. [ ] Asthma
b. [ ] Lobar pneumonia
c. [ ] Bronchopneumonia
d. [ ] Diffuse alveolar damage
e. [X] Alpha-1-antitrypsin deficiency

25
Morphopathology - CS
Tamir Bessisy 1246

209. Which of the following is true regarding viral pneumonia?


a. [X] Characterized histologically by interstitial inflammation
b. [ ] Much more common than bacterial pneumonia
c. [ ] Characterized by intraalveolar accumulation of lymphocytes
d. [ ] Characterized by necrotizing granulomas
e. [ ] None of the above
210. Lung abscess may occur:
a. [ ] As a complication of bacterial pneumonia
b. [ ] As a result of aspiration of material from carious teeth
c. [ ] As a result of viral pneumonia
d. [X] a + b
e. [ ] a + c
211. The etiologic agent common to both chronic bronchitis and emphysema is:
a. [ ] Hypersensitivity to inhaled antigens
b. [ ] Persistent viral infection
c. [ ] Asbestos
d. [X] Cigarette smoke
e. [ ] Carbon dust
212. The most common cause of community-acquired pneumonia is:
a. [X] Streptococcus pneumoniae
b. [ ] Mycoplasma pneumoniae
c. [ ] Haemophilus influenzae
d. [ ] Staphylococcus aureus
e. [ ] Mycobacterium tuberculosis
213. Which disease is matched with a characteristic histologic finding?
a. [ ] Asthma: granulomas
b. [ ] Tuberculosis: mucous gland hypertophy
c. [ ] Chronic bronchitis: eosinophils
d. [X] Emphysema: dilated acini
e. [ ] Acute respiratory distress syndrome: mucus plugging
214. Which disease is a restrictive lung disease?
a. [ ] Emphysema
b. [ ] Bronchitis
c. [X] Pneumoconiosis
d. [ ] Bronchiectasis
e. [ ] Asthma
215. Which of the following is commonly associated with lobar pneumonia?
a. [X] Acute infectious-allergic disease which involves one or more pulmonary lobe.
b. [ ] Pulmonary parenchyma acute inflammation.
c. [ ] Bronchial acute inflammation.
d. [ ] Chronic interstitial inflammation.
e. [ ] Chronic inflammation which involves one pulmonary lobe.
216.Which of the following is likely to the second stage of the lobar pneumonia.
a. [ ] Low elasticity, gray color.
b. [ ] Red colored, flaccid firmness.
c. [ ] Big patched lung.
d. [X] Red colored, increased firmness.
e. [ ] Gray colored, increased firmness.

26
Morphopathology - CS
Tamir Bessisy 1246

217. Complication of necrotizing bronchopneumonia include all of the following, EXCEPT:


a. [X] Chronic bronchitis.
b. [ ] Bronchiectasis.
c. [ ] Pleural fibrosis.
d. [ ] Metastatic abscess formation.
e. [ ] Permanent lobar solidification.
218. All of the following factors commonly predispose to bacterial pneumonia, EXCEPT:
a. [ ] Viral respiratory tract infections.
b. [ ] Cigarette smoking
c. [ ] Congestive heart failure.
d. [X] Bacterial urinary tract infection.
e. [ ] General anesthesia.
219. Bronchiectasis in adults are most often associated with:
a. [ ] Bronchial obstruction.
b. [ ] Bronchial infection.
c. [X] Both.
d. [ ] Neither.
e. [ ] Cancer.
220. The lung cancer which most commonly produces and secretes hormone-like substances is:
a. [X] Adenocarcinoma.
b. [ ] Large cell undifferentiated carcinoma.
c. [ ] Small cell undifferentiated carcinoma.
d. [ ] Squamos cell carcinoma.
e. [ ] Bronchoalveolar carcinoma.
221. Which of the chronic interstitial pneumonia listed below is characterized by marked proliferation and
desquamation of alveolar lining cells?
a. [ ] Usual interstitial pneumonitis
b. [ ] Idiopathic pulmonary fibrosis.
c. [X] Desquamative interstitial pneumonitis.
d. [ ] Lymphoid interstitial pneumonitis
e. [ ] Hamman - Rich syndrome.
222. Which of the following carcinomas grows as well - differentiated cells that line the respiratory
airspace without invading the stroma of the lung?
a. [ ] Squamos cell.
b. [ ] Anaplastic.
c. [ ] Large cell.
d. [ ] Small cell.
e. [X] Bronchioalveolar carcinoma
223. Squamous cell carcinoma of the lung is characterized by each of the following EXCEPT:
a. [ ] It is found predominantly in the major bronchi
b. [ ] It grows slower than oat cell carcinoma
c. [ ] It arises from metaplastic bronchial epithelium
d. [X] At the time of diagnosis metastases are widespread and a cure can be achieved only by chemotherapy
e. [ ] Its tumor cells are readily found in the sputum
224. All the following statements about lung cancer are true EXCEPT:
a. [ ] It is one of the most common tumors in the United States
b. [ ] Has a high mortality rate
c. [X] Has recently become more common in females than males
d. [ ] It is related to smoking
e. [ ] It is to a large extent a preventable form of cancer

27
Morphopathology - CS
Tamir Bessisy 1246

225. Lung adenocarcinomas typically


a. [ ] Metastasize rarely
b. [ ] Tend to grow more slowly than squamous cell carcinomas
c. [X] Have a propensity to invade the pleura
d. [ ] Are unrelated to cigarette smoking
e. [ ] Have decreased in frequency over the last 20 years
226. The most common malignant neoplasm of the lung is:
a. [ ] Carcinoid tumor
b. [ ] Squamous cell carcinoma of the bronchus
c. [X] Metastatic carcinoma
d. [ ] Adenocarcinoma of the bronchus
e. [ ] Oat cell carcinoma
227. All of the following are true about chronic bronchitis except that it is:
a. [ ] Primarily related to cigarette smoking
b. [ ] Defined as chronic sputum production
c. [ ] Related to the hypersecretion of mucus in response to chronic injury
d. [ ] Characterized by hyperplasia and hypertrophy of mucus-secreting apparatus
e. [X] Often caused by alpha-1-antitrypsin deficiency
228. Which of the following lesions of the lung contain numerous eosinophils?
a. [ ] Fibrosing alveolitis
b. [ ] Sarcoidosis
c. [ ] Asbestosis
d. [ ] Wegener's granulomatosis
e. [X] Bronchial asthma
229. Pulmonary hypertension is a consequence of all of the following EXCEPT:
a. [ ] Idiopathic hypoventilation
b. [ ] Recurrent pulmonary emboli
c. [ ] Congenital left-to-right shunts
d. [X] Lobar pneumonia
e. [ ] Left ventricular failure
230.Which of the following may be associated with asthma?
a. [X] The ingestion of aspirin
b. [ ] Tobacco smoke
c. [ ] The inhalation of asbestos
d. [ ] The inhalation of talc
e. [ ] Acute silicosis
231. All of the following are typical for primary tuberculosis EXCEPT:
a. [ ] It is asymptomatic.
b. [ ] It is associated with the development of immunity and hypersensitivity to proteins of M. tuberculosis.
c. [ ] It heals with the formation of a Ghon complex.
d. [ ] The most serious immediate complication is miliary tuberculosis.
e. [X] The majority of patients develop primary progressive tuberculosis
232. Familial emphysema is usually due to a defect in
a. [ ] The structure and function of cilia
b. [X] Circulating antiprotease activity
c. [ ] Epithelial chloride transport
d. [ ] Regulation of immunoglobulin E
e. [ ] Pulmonary surfactant

28
Morphopathology - CS
Tamir Bessisy 1246

233. The ethiologic agent common to chronic bronchitis and emphysema is


a. [ ] A persistant viral infection
b. [ ] Hypersensitivity to inhaled allergens
c. [X] Tobacco smoke
d. [ ] Asbestos
e. [ ] Alcohol
234.Charcot-Leyden crystals occur in:
a. [ ] Bronchiolitis obliterans
b. [ ] Centrilobular emphysema
c. [ ] Chronic bronchitis
d. [X] Asthma
e. [ ] Bronchiectasis
235. All of the following are true about bronchial asthma EXCEPT:
a. [ ] Changes in both lungs
b. [ ] Curschmann's spirals and Charcot-Leyden crystals in sputum
c. [X] Prone to develop lung abscess
d. [ ] Mediated by leukotrienes
e. [ ] Mucus in bronchi
236. All of the following are true about chronic bronchitis except that it is:
a. [ ] Primarily related to cigarette smoking
b. [ ] Defined as chronic sputum production
c. [ ] Related to the hypersecretion of mucus in response to chronic injury
d. [ ] Characterized by hyperplasia and hypertrophy of mucus-secreting apparatus
e. [X] Often caused by alpha-1-antitrypsin deficiency
237. Squamous cell carcinoma of the lung is associated with each of the following except:
a. [X] Peripheral location
b. [ ] Intercellular bridges
c. [ ] Pancoast tumor
d. [ ] Central cavitation
e. [ ] Cytokeratin filaments
238.All of the following are true of hyaline membranes in the lung EXCEPT:
a. [ ] Occur in newborns and adults
b. [ ] May be the result of oxygen toxicity
c. [ ] Occur in shock
d. [ ] Consist of fibrin
e. [X] Contain immune complexes
239.In typical acute bacterial pneumonia, the alveolar spaces are most likely to contain:
a. [ ] Plasma cells and fibrin
b. [ ] Lymphocytes and fibrin
c. [ ] Macrophages and hemorrhage
d. [X] Polymorphonuclear leukocytes and fibrin
e. [ ] Macrophages and fibrin
240. True statements regarding pulmonary embolism and infarction include all of the following, EXCEPT:
a. [ ] Occlusion of pulmonary arteries by blood clots is usually embolic in nature
b. [ ] The usual source of pulmonary emboli is the deep veins of the legs
c. [X] When emboli reach the lungs, infarction typically occurs in 80-90% of cases
d. [ ] Saddle emboli cause sudden death by blockage of pulmonary blood flow
e. [ ] Unresolved multiple small emboli over time may lead to pulmonary hypertension
241. Which of the following may involve extrapulmonary tuberculosis?
a. [ ] Adrenal

29
Morphopathology - CS
Tamir Bessisy 1246

b. [ ] Fallopian tube
c. [ ] Lymph node
d. [ ] Pleura
e. [X] All of the above
242. Which of the following is a recognized contributing cause of death in a patient with severe pulmonary
emphysema?
a. [ ] Respiratory acidosis
b. [ ] Acute intercurrent bacterial infection
c. [ ] Right sided heart failur
d. [ ] Severe pneumothorax
e. [X] All of the above
243. Which of the following statements best describes compression atelectasis:
a. [ ] Consequence of complete obstruction of an airway
b. [ ] The mediastinum characteristically shifts toward the atelectatic lung
c. [X] Consequence of filling of the pleural space with fluid or air
d. [ ] Results from loss of pulmonary surfactant
e. [ ] Consequence of local or generalized fibrotic changes in the lung
244. True statements regarding hyaline membranes and diffuse alveolar injury include all of the following, EXCEPT:
a. [ ] Hyaline membranes consist of fibrin-rich edema fluid and necrotic cell debris
b. [ ] The presence of hyaline membranes reflects diffuse alveolar injury
c. [X] Hyaline membranes in premature infants are filled with lymphocyte
d. [ ] Hyaline membranes arise from alveolar injury due to a variety of insults
e. [ ] Diffuse alveolar injury in ARDS and in the respiratory distress syndrome of newborns
arise by different pathogenic mechanisms
245. Pneumonia with microabscess formation is most likely due to which of the following organisms?
a. [ ] Beta hemolytic streptococcus
b. [ ] Klebsiella pneumoniae
c. [ ] Legionella pneumophila
d. [ ] Pseudomonas aeroginosa
e. [X] Staphylococcus aureus
246. Bronchioloalveolar carcinoma:
a. [ ] Accounts for 30% of lung carcinomas
b. [ ] Is characterized histologically by a solid pattern of spread
c. [ ] The mucinous/diffuse variety is associated with a better prognosis
d. [X] The non-mucinous variant may arise from Clara cells or type II pneumocytes
e. [ ] Develop from the central bronchi
247. Which of the following statements is most correct regarding pulmonary edema:
a. [ ] The primary determinant of colloid osmotic pressure within pulmonary capillaries is the concentration of Na+ ions
b. [ ] High altitude pulmonary edema is an example of neurogenic edema
c. [ ] Injury to pulmonary vasculature is uncommon due to the unique properties of the endothelial cells therein
d. [X] Left-sided congestive heart failure leads to pulmonary edema through increased hydrostatic pressure
e. [ ] Hemosiderin-laden macrophages usually indicate the presence of acute pneumonia
248. Which of the following is the pathogenetic mechanism that leads to the development of lung abscess?
a. [ ] Aspiration of infected material
b. [ ] Complication of bacterial pneumonia
c. [ ] Secondary infection of a pulmonary infarct
d. [ ] Septic embolization from an infection in another organ
e. [X] All of the above

30
Morphopathology - CS
Tamir Bessisy 1246

249.Which is TRUE regarding small cell carcinoma of lung?


a. [ ] It is not associated with smoking
b. [ ] Most commonly peripheral in location.
c. [X] Excellent response to chemotherapy
d. [ ] Overall prognosis (5-year survival) is in the range of 30-40%
e. [ ] It is well differentiated
250. The characteristic lesion in primary pulmonary tuberculosis is:
a. [ ] Cavitary lesion in the lung apex
b. [X] Ghon's complex
c. [ ] Localized bronchiectasis
d. [ ] Miliary lesions
e. [ ] Tuberculous pneumonia
251. Carcinoid tumors characteristically display:
a. [ ] Immunoperoxidase stains negative for NSE, chromogranin and synaptophysin
b. [X] Electron microscopy showing dense core electron dense granules
c. [ ] Immunoperoxidase stains positive for HMB45 and S-100
d. [ ] Electron microscopy showing melanosomes
e. [ ] Immunoperoxidase stains positive for CD3 and CD20
252. Which of the following is a cause of pneumothorax?
a. [ ] Chest trauma
b. [ ] Emphysema
c. [ ] Lung abscess
d. [ ] Needle biopsy of the pleura
e. [X] All of the above
253. Regarding squamous cell carcinoma (SCC) of the lung, which of the following is TRUE?
a. [ ] SCC is a peripheral tumor
b. [ ] SCC has a strong association with scarring (scar cancer)
c. [X] Is characterized histologically by keratin formation and intercellular bridges
d. [ ] A paraneoplastic syndrome due to ectopic ACTH production is associated with SCC
e. [ ] Caseous necrosis is present.
254. Which of the following is a characteristic histologic finding in asthma:
a. [ ] Attenuation of the bronchial submucous gland layer
b. [ ] Attenuation of the basement membrane of bronchial epithelium
c. [ ] Mixed inflammatory infiltrate in the bronchial walls, with a predominance of mast cells
d. [X] Hypertrophy of bronchial wall muscle
e. [ ] Destruction of alveolar walls
255. Which of the following statements regarding centriacinar emphysema is TRUE?
a. [X] It involves the respiratory bronchioles
b. [ ] It occurs predominantly in female smokers
c. [ ] It affects the right lung more severely than the lef
d. [ ] It is often associated with chronic alcoholism
e. [ ] It is usually seen in drug addicts
256. All of the following are complications of severe bronchial asthma EXCEPT:
a. [ ] Bronchiectasis
b. [ ] Pneumonia
c. [ ] Chronic bronchitis
d. [ ] Cor pulmonale
e. [X] Bronchogenic carcinoma

31
Morphopathology - CS
Tamir Bessisy 1246

257. Viral infections of the lung most commonly cause:


a. [ ] Acute airspace pneumonia with accumulation of PMN
b. [ ] Aspiration pneumonia and accumulation of PMN
c. [X] Interstitial pneumonia with predominantly lymphocytic reaction
d. [ ] Granulomatous pneumonia and mast cell granulation
e. [ ] Opportunistic pneumonia and eosinophil accumulation
258. The most common cause of intestinal obstruction is:
a. [ ] Volvulus
b. [ ] Neoplasm
c. [ ] Intussusception
d. [X] Hernia
e. [ ] Adhesions
259. A two-week-old boy develops persistent projectile vomiting. The most likely diagnosis is:
a. [X] Pyloric stenosis
b. [ ] Esophageal atresia
c. [ ] Annular pancreas
d. [ ] Incomplete rotation of the gut
e. [ ] Amyloidosis
260. An endoscopic biopsy of gastric mucosa reveals small intestinal type epithelium This finding is most likely due to:
a. [X] Chronic gastritis
b. [ ] Congenital heterotopia
c. [ ] Precancerous dysplasia
d. [ ] Metastatic carcinoma
e. [ ] Benign neoplasm
261. Hematemesis is an indication of:
a. [X] Upper gastrointestinal bleeding
b. [ ] Lower gastrointestinal bleeding
c. [ ] Middle gastrointestinal bleeding
d. [ ] All of the above
e. [ ] None of the above
262. The Mallory-Weiss syndrome refers to the occurrence of gastrointestinal hemorrhage as a result of:
a. [ ] Esophageal varices
b. [ ] Esophageal peptic ulcers
c. [X] Mucosal tears at the esophagogastric junction
d. [ ] Iatrogenic perforation
e. [ ] Ulcerogenic drugs
263. Primary carcinoma is least common in:
a. [ ] Esophagus
b. [ ] Stomach
c. [X] Small intestine
d. [ ] Colon
e. [ ] Rectum
264. Bilateral ovarian metastases presenting as tumor masses are most characteristically associated with
carcinoma of the:
a. [ ] Esophagus
b. [X] Stomach
c. [ ] Small intestine
d. [ ] Appendix
e. [ ] Colon

32
Morphopathology - CS
Tamir Bessisy 1246

265. What is the most common cause of esophageal varices?


a. [X] Alcoholic cirrhosis
b. [ ] Cardiac cirrhosis
c. [ ] Extra-hepatic portal vein obstruction
d. [ ] Esophagitis
e. [ ] All of the above
266. Which is characterized by vomiting of blood?
a. [ ] Zenker’s diverticulum
b. [ ] Traction diverticulum
c. [ ] Achalasia
d. [X] Mallory-Weiss syndrome
e. [ ] Hiatal hernia
267. Acute erosive gastritis is characterized by:
a. [ ] Pus in the stomach
b. [X] Superficial multiple ulcerations of gastric mucosa
c. [ ] A deep ulcer of the stomach with a scarred base
d. [ ] A frequent association with gastric cancer
e. [ ] Perforation as frequent complication
268. The most frequent complication of chronic duodenal ulcer is:
a. [X] Hemorrhage
b. [ ] Obstruction
c. [ ] Perforation
d. [ ] Malabsorption
e. [ ] All of the above
269. Carcinoma of the stomach usually arises from:
a. [ ] Smooth muscle cells
b. [ ] Acid producing cells
c. [ ] Pepsinogen producing cells
d. [X] Mucus producing cells
e. [ ] Argentaffin cells
270. Which feature of ulcerative colitis is most closely related to the development of carcinoma?
a. [ ] Age of patient at time of onset
b. [X] The duration of the disorder
c. [ ] Severity of manifestations at onset
d. [ ] Presence of pseudopolyps
e. [ ] Presence of melena
271. The most typical distribution of the inflammatory process in ulcerative colitis is:
a. [ ] Entire colon and terminal ileum
b. [ ] Diffuse, involving entire colon
c. [ ] Focal, segmental
d. [X] Rectum and a variable length of contiguous colon
e. [ ] Stomach and duodenum
272. Which neoplasm is most FREQUENTLY found in the appendix?
a. [X] Carcinoid
b. [ ] Villous adenoma
c. [ ] Lymphoma
d. [ ] Adenomatous polyp
e. [ ] Adenocarcinoma

33
Morphopathology - CS
Tamir Bessisy 1246

273. Colonic neoplasms tend to metastasize most frequently to:


a. [X] Liver
b. [ ] Lung
c. [ ] Vertebral column
d. [ ] Small intestine
e. [ ] Kidney
274. In contrast to carcinoma of the right colon, carcinoma of the left colon tends to be associated with:
a. [ ] Anemia
b. [ ] Diverticulosis
c. [ ] Malabsorption
d. [X] Obstruction
e. [ ] No symptoms
275. The most common fatal complication of chronic peptic ulcer of the stomach is:
a. [ ] Adenocarcinoma
b. [ ] Acute gastritis
c. [X] Perforation and peritonitis
d. [ ] Pancreatitis
e. [ ] Pyloric outlet obstruction
276. Which of the following types of esophagitis is the most common?
a. [X] Reflux
b. [ ] Viral
c. [ ] Fungal
d. [ ] Acute corrosive
e. [ ] Chronic granulomatous
277. The most common cause of the peritonitis are the following EXCEPT:
a. [ ] Gastric ulcer perforation
b. [ ] Intestinal perforation in typhoid fever
c. [ ] Gangrenous appendicitis
d. [X] Focal pneumonia
e. [ ] Acute pancreatitis
278. Characteristic gross features of a benign gastric ulcer include all of the following EXCEPT:
a. [ ] Location on the lesser curvature
b. [ ] Small size
c. [X] Heaped-up margins
d. [ ] Smooth base
e. [ ] Radial arrangement of surrounding mucosal folds
279. Features of colonic adenomas that are associated with increased risk for carcinoma include all of
the following EXCEPT:
a. [ ] Severe dysplasia
b. [ ] Villous architecture
c. [ ] Size exceeding 2 cm
d. [X] Marked inflammation
e. [ ] Multiple adenomas
280. Which of the following inflammatory conditions of the intestine is characterized by segmental involvement
of the small/or large bowel, transmural inflammation, and the development of epithelioid granulomas?
a. [X] Crohn's disease
b. [ ] Ulcerative colitis
c. [ ] Cryptosporidiosis
d. [ ] Diverticulitis
e. [ ] Colitis cystica profunda

34
Morphopathology - CS
Tamir Bessisy 1246

281. The most common site of gastrointestinal carcinoid tumors is:


a. [ ] Small bowel
b. [ ] Colon
c. [X] Appendix
d. [ ] Esophagus
e. [ ] Stomach
282. The cause of ulcerative colitis in most cases is
a. [ ] Psychosomatic
b. [ ] Viral
c. [ ] Autoimmune
d. [ ] Bacterial
e. [X] Undetermined
283. Which of the following is associated with an increased risk of esophageal cancer?
a. [ ] Herpetic esophagitis
b. [ ] Candida esophagitis
c. [X] Cigarette smoking
d. [ ] Mallory-Weiss syndrome
e. [ ] Boerhave syndrome
284. Acute gastric erosions occur in each the following settings EXCEPT
a. [ ] Extensive burns
b. [ ] Alcohol abuse
c. [ ] Trauma to the brain
d. [ ] Irreversible shock
e. [X] Pernicious anemia
285. Which of the following bowel disease is associated with arthritis and uveitis?
a. [X] Crohn disease
b. [ ] Pseudomembranous colitis
c. [ ] Mycobacterium avium intracellulare enterocolitis
d. [ ] CMV colitis
e. [ ] Vibrio cholera
286. The most common symptom or complication of peptic ulcer in the duodenum is:
a. [X] Bleeding
b. [ ] Malignant transformation
c. [ ] Rupture
d. [ ] Peritonitis
e. [ ] Obstruction
287. Pernicious anemia is usually associated with:
a. [ ] Gastric hypersecretion
b. [X] Gastric adenocarcinoma
c. [ ] Hypertrophic gastritis
d. [ ] Increased insulin levels
e. [ ] Autoantibodies to the intrinsic factor
288. Familial polyposis coli is characterized by each of the following EXCEPT:
a. [ ] Multiple tubular adenomas
b. [ ] Autosomal dominant inheritance
c. [X] Colonic polyps usually are present at birth
d. [ ] High incidence of malignancy
e. [ ] Multiple villous adenomas

35
Morphopathology - CS
Tamir Bessisy 1246

289. Carcinoma of the oral cavity is most often:


a. [X] Squamous
b. [ ] Basal cell
c. [ ] Transitional cell
d. [ ] Ameloblastic
e. [ ] Odontogenic
290. Acute gastric ulceration may be associated with each the following conditions EXCEPT:
a. [ ] Extensive burns
b. [ ] Cerebrovascular accidents
c. [ ] Corticosteroid therapy
d. [ ] Excessive alcohol intake
e. [X] Achalasia of esophagus
291. Anemia associated with gastric carcinoma is usually a result of:
a. [X] Prolonged blood loss
b. [ ] Metastases to bone
c. [ ] Metastases to the small intestine
d. [ ] Spread of tumor to esophagus
e. [ ] Vitamin B12 deficiency
292. Each of the following is associated with squamous cell carcinoma of the esophagus EXCEPT:
a. [ ] Alcoholism
b. [X] Hot and spicy foods
c. [ ] Chronic esophagitis
d. [ ] Achalasia
e. [ ] Cigarette smoking
293. Which of the following pathologic features is characteristic of squamous cell carcinoma of the esophagus?
a. [ ] Well differentiated glands
b. [ ] Arises in zones of metaplastic mucosa called Barrett esophagus
c. [X] Clinical symptomsus ually appear only when the tumor is advanced beyond the point of curative resection
d. [ ] Mostly occurs in conjunction with esophageal varices
e. [ ] Early distant metastases are common, whereas lymphatic spread to regional lymph nodes is rare
294. Each of the following is true of gastric leiomyomas EXCEPT:
a. [X] They are more common than gastric adenocarcinoma
b. [ ] They may cause erosion and bleeding if larger than 2 cm
c. [ ] They are usually benign
d. [ ] As a group, they may show a variety of histologic patterns
e. [ ] Histologic criteria for malignancy include mitotic frequency
295. Each the following statements about gastric carcinoma are true EXCEPT:
a. [ ] Incidence rates of gastric carcinoma show wide geographic differences
b. [X] Prognosis in gastric carcinoma is related to stage of disease
c. [ ] Absent acid secretion after pentagastrin stimulation is highly correlated with gastric carcinoma
d. [ ] Rarely, if ever, found in blood group A persons
e. [ ] Many tumors are mucin producing
296. Which of the following types of polyps is not neoplastic?
a. [ ] Tubular adenoma
b. [ ] Adenomatous polyp
c. [X] Juvenile polyp
d. [ ] Villous adenoma
e. [ ] Polypoid leiomyoma

36
Morphopathology - CS
Tamir Bessisy 1246

297. So called "acute gastritis" is best described by what the pathologist sees In this sense, it is best termed:
a. [ ] Hypertrophic gastritis
b. [ ] Hyperplastic gastritis
c. [X] Erosive gastritis
d. [ ] Atrophic gastritis
e. [ ] Non-necrotizing granulomatous gastritis
298. Non-erosive gastritis can lead to each of the following EXCEPT:
a. [ ] Achlorhydria
b. [ ] Pernicious anemia
c. [ ] Gastric atrophy
d. [ ] Gastric carcinoma
e. [X] Foliate deficiency anemia
299. Pseudomembranous colitis is a:
a. [ ] Granulomatous inflammation that is caused by Camplylobacter, and rarely shows transmural involvement
b. [ ] Non-necrotizing granulomatous inflammatory condition of the colon which is of unknown etiology
c. [X] Bacterial infection of the colon characterized by focal mucosal ulceration and the formation of
fibrinomucinous exudate over denuded areas
d. [ ] Transmural inflammatory condition that is characterized by focal granulomatous inflammatory infiltrates
that are best seen along the serosal surface of the colon
e. [ ] Disease not associated with any known organism and is characterized by focal mucosal ulceration with
the formation of a fibrinous exudate over denuded regions
300. Which of the following is true of ulcerative colitis?
a. [ ] The inflammation is frequently transmural and may produces serosal adhesions
b. [ ] The inflammation is granulomatous in nature
c. [ ] It frequently presents as a mass lesion
d. [ ] Skip lesion are usually apparent on colonoscopy
e. [X] In the acute stage a biopsy might look identical to Shigella infection
301. Which of the following is true of Crohn disease?
a. [X] The disease shows transmural inflammation
b. [ ] The rectum is almost always involved
c. [ ] Skip lesions are not seen
d. [ ] Similar histologic finding are seen in infections with Vibrio cholera
e. [ ] The disease frequently presents in the sixth and seventh decades of life
302. The Mallory-Weiss syndrome is:
a. [ ] Associated with iron deficiency anemia
b. [ ] Is seen in the upper esophagus
c. [X] Is a laceration of the esophagus caused by severe retching
d. [ ] Is a perforation of the superior aspect of the stomach caused by chemical injury
e. [ ] Is caused by a motor dysfunction in the esophagus
303. Which of the following sequences describes the arrangement of the histologically distinct layers,
starting from the luminal side of the intestine toward outside?
a. [ ] Fibrosis, acute inflammation, necrosis, chronic inflammation
b. [X] Necrosis, acute inflammation, chronic inflammation, fibrosis
c. [ ] Acute inflammation, fibrosis, chronic inflammation, necrosis
d. [ ] Necrosis, fibrosis, acute inflammation, chronic inflammation
e. [ ] None of the above
304. Barrett esophagus refers to:
a. [ ] A type of esophageal diverticulum
b. [X] Glandular metaplasia of the esophageal mucosa
c. [ ] Congenital presence of glandular epithelium in the esophageal mucosa

37
Morphopathology - CS
Tamir Bessisy 1246

d. [ ] A lesion which gives rise to esophageal varices


e. [ ] A lesion which, once established, is not reversible
305. Which of the following tumors of the esophagus is the most common?
a. [X] Squamous cell carcinoma
b. [ ] Adenocarcinoma
c. [ ] Lymphoma
d. [ ] Metastatic tumors
e. [ ] Leiomyosarcoma
306. Which of the following cell types is NOT ordinarily found in the gastric mucosa?
a. [ ] Chief cells
b. [ ] Parietal cells
c. [ ] Enterochromaffin cells
d. [X] Paneth cells
e. [ ] All of the above
307. All of the following is positively associated with gastric carcinoma EXCEPT:
a. [ ] Atrophic gastritis
b. [ ] Pernicious anemia
c. [ ] Peptic ulceration of stomach by x-ray
d. [X] Gastrinoma
e. [ ] Intestinal metaplasia
308. Which part of the GI tract vitamin B12 is principally absorbed in?
a. [ ] Duodenum
b. [ ] Jejunum
c. [X] Ileum
d. [ ] Cecum
e. [ ] Colon
309. Inflammatory bowel disease is associated with which of the following terms?
a. [ ] Ulcerative colitis
b. [ ] Crohn disease
c. [ ] Regional enteritis
d. [ ] Toxic megacolon
e. [X] All of the above
310. Which of the following diseases is marked by an increased risk of colon carcinoma?
a. [ ] Congenital megacolon
b. [ ] Acquired megacolon
c. [ ] Pseudomembranous colitis
d. [X] Chronic ulcerative colitis
e. [ ] Angiodysplasia of the colon
311. The most common benign mesenchymal tumor in the gastrointestinal tract is:
a. [ ] Angioma
b. [X] Leiomyoma
c. [ ] Neuroma
d. [ ] Adenoma
e. [ ] Inflammatory pseudopolyp
312. Which of the following is a cause of melena?
a. [ ] Acute appendicitis
b. [X] Duodenal ulcer
c. [ ] Hemorrhoids
d. [ ] Meckel's diverticulum
e. [ ] All of the above

38
Morphopathology - CS
Tamir Bessisy 1246

313. All of the following statements are true of peptic ulcers, EXCEPT:
a. [ ] Bicarbonate and mucus are gastric defence factors
b. [ ] Most common site is duodenum or gastric antrum
c. [X] Most often are multiple lesions
d. [ ] Most individuals with peptic ulcers are infected with Helicobacter pylori
e. [ ] Bleeding is the most frequent complication
314. Each of the following is a true statement regarding Crohn's disease (CD) or ulcerative colitis (UC), EXCEPT:
a. [ ] CD may involve any portion of the gastrointestinal tract
b. [X] Fistula formation is characteristic of advanced UC
c. [ ] The risk of carcinoma is considered greater in UC than in CD
d. [ ] CD typically involves some portions of the bowel, but skips adjacent portions
e. [ ] Pseudopolyps are characteristic of UC
315. Which of the following statements best describes hyperplastic polyps of the colon?
a. [X] Tend to be small (less than 5 mm in diameter)
b. [ ] Composed of basophilic cells with irregular nuclei
c. [ ] Most commonly identified in children under the age of 10
d. [ ] Significant malignant potential
e. [ ] Usually have a long stalk (peduncle)
316. Chronic gastritis is:
a. [X] Recognized consequence of chronic infection with H. pylori
b. [ ] Tortuous dilatations of submucosal and mucosal blood vessels in the colon
c. [ ] Variceal dilatations of the anal venous plexus
d. [ ] Telescoping of one intestinal segment into another
e. [ ] Failure of the cloacal diaphragm to open during development
317. Gastric cancer Virchow metastasis is:
a. [ ] Retroperitoneal lymph nodes
b. [ ] Ovaries
c. [ ] Lymph glands
d. [ ] Thoracic lymph nodes
e. [X] Supraclavicular lymph nodes
318. Typhoid fever source of infection may be:
a. [ ] Cootie
b. [ ] Mite
c. [ ] Dog
d. [ ] Pig
e. [X] A sick person
319. The second stage of typhoid fever is characterized by:
a. [ ] Typhoid granuloma necrosis
b. [ ] Follicles group necrosis
c. [ ] Solitary follicles necrosis
d. [ ] Intramural nerve ganglia dystrophy
e. [X] All of the above
320. Which of the following conditions can occur in diabetic patients:
a. [ ] Osmotic diuresis
b. [ ] Nephrotic syndrome
c. [ ] Trophic big toe ulcer
d. [ ] Retinal hemorrhage
e. [X] All of the above

39
Morphopathology - CS
Tamir Bessisy 1246

321. Which of the following lesions may be detected in diabetes mellitus:


a. [ ] Pancreatic amyloid deposits
b. [ ] Renal tubular vacuolization
c. [ ] Glomerulosclerosis
d. [ ] Arteriolosclerosis
e. [X] All of the above
322. Which one of the following is not included in the manifestation of Grave’s disease?
a. [X] Thyroiditis
b. [ ] Thyroitoxicosis
c. [ ] Infiltrative ophthalmopathy
d. [ ] Tachycardia
e. [ ] Localized infiltrative dermopathy
323. A complex Syndrome resulting from the absence or deficiency of the puritary hormone:
a. [X] Hypopituitarism
b. [ ] Dwarfism
c. [ ] Acromegaly
d. [ ] Gigantism
e. [ ] Nanism
324. A proportional overgrowth of the body’s tissue due to the hypersecretion of the human growth
hormone before puberty:
a. [ ] Hypopituitarism
b. [X] Gigantism
c. [ ] Acromegaly
d. [ ] Dwarfism
e. [ ] Hyperparathyroidism
325. Under –section of thyroxine hormone in babies – present at birth causing mental and physical retardation
a. [X] Cretinism
b. [ ] Dwarfism
c. [ ] Gigantism
d. [ ] Tetany
e. [ ] Acromegaly
326. Abnormal protrusion of the eyeball
a. [ ] Tetany
b. [X] Exophthalmos
c. [ ] Gigantism
d. [ ] Myxoedema
e. [ ] Endophtalmos
327. Excessive production of parathyroid hormone from parathyroid glands
a. [ ] Hypersecretion
b. [X] Hyperparathyroidism
c. [ ] Hypoparathyroidism
d. [ ] Hyposecretion
e. [ ] All of the above
328. Excessive adrenal cortex hormones causing abnormal distribution of hair, fat and shrinking of genitals
a. [ ] Cretinism
b. [X] Cushing's syndrome
c. [ ] Malignant goitre
d. [ ] Addison's disease
e. [ ] Kuhn syndrome

40
Morphopathology - CS
Tamir Bessisy 1246

329. An ACTH stimulation test is commonly used to diagnose:


a. [ ] Grave's disease
b. [X] Adrenal insufficiency and Addison's disease
c. [ ] Cystic fibrosis
d. [ ] Hashimoto's disease
e. [ ] Kohn disease
330. The most common benign tumor of the pituitary gland is a:
a. [ ] Glioma
b. [X] Prolactinoma
c. [ ] Carcinoid tumor
d. [ ] Islet cell tumor
e. [ ] Astrocytoma
331. Women with polycystic ovarian syndrome (PCOS) are at increased risk for all of the following except:
a. [X] Pregnancy
b. [ ] Diabetes
c. [ ] Cardiovascular disease
d. [ ] Metabolic syndrome
e. [ ] Osteoporosis
332. The most common causes of death in people with cystic fibrosis is:
a. [ ] Dehydration
b. [ ] Opportunistic infection
c. [ ] Lung cancer
d. [X] Respiratory failure
e. [ ] All of the above
333. Short stature and undeveloped ovaries suggest which of the following disorders:
a. [ ] Polycystic ovarian syndrome
b. [ ] Prolactinoma
c. [ ] Grave's disease
d. [X] Turner syndrome
e. [ ] Waterhouse-Friderichsen syndrome
334. What is the most common cause of hypothyroidism worldwide?
a. [ ] Autoimmune disease
b. [ ] Graves’ disease
c. [ ] Iatrogenic causes
d. [X] Iodine deficiency
e. [ ] Medication side effects
335. The hormones regulating blood calcium levels are:
a. [ ] Insulin and glucagon
b. [ ] Glycogen and pth
c. [ ] Inhibiting hormones
d. [X] Pth and calcitonin
e. [ ] Calcitonin and atp
336. A hormone that under certain circumstances is regulated by positive feedback is:
a. [ ] calcitonin
b. [ ] histamine
c. [X] oxytocin
d. [ ] melatonin
e. [ ] insulin

41
Morphopathology - CS
Tamir Bessisy 1246

337. The pituitary gland is attached to the hypothalamus by the:


a. [ ] epithalamus
b. [X] infundibulum
c. [ ] parafollicular cells
d. [ ] intermediate mass
e. [ ] corpus callosum
338. Which of the following hormones stimulates testosterone production by the testis?
a. [ ] TSH
b. [ ] FSH
c. [ ] ACTH
d. [X] LH
e. [ ] GH
339. Which of the following hormones is released in response to a nerve impulse?
a. [X] Epinephrine
b. [ ] Cortisol
c. [ ] Testosterone
d. [ ] Insulin
e. [ ] Glucagon
340. The gland which can be classified as an endocrine and an exocrine glands is the:
a. [ ] Thyroid
b. [ ] Thymus
c. [X] Pancreas
d. [ ] Pituitary
e. [ ] Hypothalamus
341. The development and maintenance of the female sex characteristics is the responsibility of:
a. [ ] Estrogen and androgen
b. [ ] Progesterone and testosterone
c. [ ] Relaxin and inhibin
d. [ ] Progesterone and relaxin
e. [X] Progesterone and estrogen
342. Which hormone stimulates cells to grow and divide?
a. [ ] Thyroid stimulating hormone
b. [ ] Luteinizing hormone
c. [X] Growth Hormone
d. [ ] Gluccocorticoids
e. [ ] Insulin
343. Insulin is secreted by the pancreas after a meal in order to:
a. [X] Decrease the concentration of blood glucose
b. [ ] Decrease the permeability of the cell membranes to glucose
c. [ ] Increase the production of glucose from glycogen
d. [ ] Increase the concentration of blood glucose
e. [ ] Increase the amount of thyroid hormones in the blood
344. Simmonds' disease is caused by the following disorders:
a. [ ] Thyroid
b. [ ] Pancreas
c. [ ] Adrenal
d. [X] Pituitary
e. [ ] Epiphysis

42
Morphopathology - CS
Tamir Bessisy 1246

345. Addison’s disease is caused by:


a. [ ] Hyperthyroidism
b. [ ] Hypothyroidism
c. [ ] Adrenal hyperfunction
d. [X] Adrenal hypofunction
e. [ ] Parathyroid gland hyperfunction
346. Hydropic swelling results from:
a. [ ] Membrane rupture
b. [ ] ATP accumulation
c. [ ] Oncogene activation
d. [X] Na+/K+ pump dysfunction
e. [ ] Cytoplasm lisis
347. Coagulative necrosis:
a. [ ] Resemble crumbly cheese
b. [X] Can result from interrupted blood supply
c. [ ] Is reversible if promptly and aggressively managed
d. [ ] Remains functional for 5-7 days
e. [ ] Affects only extremities
348. Apoptosis is a process that results in:
a. [ ] Cellular atrophy
b. [X] Cellular death
c. [ ] Cellular proliferation
d. [ ] Cellular mutation
e. [ ] Cellular dysplasia
349. Necrotic death of brain tissue usually produces what type of necrosis?
a. [ ] Coagulative
b. [ ] Caseous
c. [X] Liquefactive
d. [ ] Fat
e. [ ] Gangrene
350. Which of the following cellular responses is indicative of injury due to faulty metabolism?
a. [ ] Hydropic swelling
b. [ ] Lactate production
c. [ ] Metaplasia
d. [X] Intracellular accumulations
e. [ ] Hypertrophy
351. A high serum lactate level (lactic acidosis) usually indicates the presence of:
a. [ ] Liver failure
b. [ ] Hypoglycemia
c. [ ] Immunologic injury
d. [X] Cellular hypoxia
e. [ ] Hypocalcemia
352. Ischemic death of tissue in visceral organs, such as the heart, typically produces:
a. [X] Coagulative necrosis
b. [ ] Liquefactive necrosis
c. [ ] Caseous necrosis
d. [ ] Fat necrosis
e. [ ] Proteic necrosis

43
Morphopathology - CS
Tamir Bessisy 1246

353. We remember Rudolf Virchow LEAST for:


a. [ ] Being the founder of modern-day pathology
b. [ ] His idea that all cells come from pre-existing cells
c. [ ] His idea that all disease is disease of cells
d. [ ] His idea that politics is a major cause of human disease
e. [X] His focus on disease at the "whole person" level
354. Which is NOT a feature of apoptotic cell death?
a. [ ] Cutting of the DNA at regular intervals
b. [ ] Cross-linking of proteins in the cytoplasm
c. [ ] Destruction of the cytoskeleton
d. [X] Liquefaction of the apoptotic remnant
e. [ ] Pyknosis of the nucleus as in other forms of necrosis
355. Which ion is blamed for "reperfusion injury"?
a. [X] Calcium
b. [ ] Magnesium
c. [ ] Phosphate
d. [ ] Potassium
e. [ ] Sodium
356. You're most likely to see caseous necrosis in:
a. [ ] Calcified fat around a wounded pancreas
b. [ ] Gangrenous diabetic foot
c. [ ] Infarcted myocardium
d. [ ] Pus in a boil
e. [ ] Ebdbd
357. Which of the following describes hyperplasia?
a. [X] Increase in the number of cells (mitosis) in an organ or tissue
b. [ ] Decrease in the number of cells (mitosis) in an organ or tissue
c. [ ] Increase in individual cell size in an organ or tissue
d. [ ] Decrease in individual cell size in an organ or tissue
e. [ ] Reversible change in which one adult cell is replaced by another adult cell type
358. Most forms of pathologic hyperplasia are caused by excessive hormonal stimulation or growth factors acting on
target cells. If a patient had hyperplasia of the endometrium, which of the following is the most likely?
a. [ ] Increased risk of miscarriage
b. [ ] Decreased risk of miscarriage
c. [X] Increased risk of endometrial cancer
d. [ ] Decreased risk of endometrial cancer
e. [ ] Increased risk of neurologic disease
359. Infection from which of the following is associated with hyperplasia?
a. [X] Papillomavirus
b. [ ] Enterobacteria
c. [ ] Staphylococci
d. [ ] Streptococci
e. [ ] Hook worms
360. Which of the following describes hypertrophy?
a. [ ] Increase in the number of cells (mitosis) in an organ or tissue
b. [ ] Decrease in the number of cells (mitosis) in an organ or tissue
c. [X] Increase in individual cell size in an organ or tissue
d. [ ] Decrease in individual cell size in an organ or tissue
e. [ ] Reversible change in which one adult cell is replaced by another adult cell type

44
Morphopathology - CS
Tamir Bessisy 1246

361. Which of the following types of atrophy is involved in ischemia?


a. [ ] Decreased workload
b. [ ] Loss of innervation
c. [X] Diminished blood supply
d. [ ] Inadequate nutrition (protein-calorie)
e. [ ] Loss of endrocrine stimulation
362. Which of the following is associated with cachexia, seen in patients with chronic inflammatory
diseases and cancer?
a. [ ] Decreased workload
b. [ ] Loss of innervation
c. [ ] Diminished blood supply
d. [X] Inadequate nutrition (protein-calorie)
e. [ ] Loss of endrocrine stimulation
363. Atrophy may be accompanied by residual bodies, such as lipofuscin granules, which can turn
tissue what color?
a. [ ] Yellow
b. [ ] Blue
c. [X] Brown
d. [ ] White
e. [ ] Red
364. Which of the following describes metaplasia?
a. [ ] Increase in the number of cells (mitosis) in an organ or tissue
b. [ ] Decrease in the number of cells (mitosis) in an organ or tissue
c. [ ] Increase in individual cell size in an organ or tissue
d. [ ] Decrease in individual cell size in an organ or tissue
e. [X] Reversible change in which one adult cell is replaced by another adult cell type
365. Which of the following is the most common stem cell reprogramming change involved in respiratory tract cancer?
a. [ ] Squamous to columnar
b. [ ] Squamous to cuboidal
c. [X] Columnar to squamous
d. [ ] Columnar to cuboidal
e. [ ] Cuboidal to squamous
366. In Barrett esophagus, metaplasia occurs as a result of refluxed gastric acid. Which of the following changes occurs?
a. [X] Squamous to columnar
b. [ ] Squamous to cuboidal
c. [ ] Columnar to squamous
d. [ ] Columnar to cuboidal
e. [ ] Cuboidal to squamous
367. Which of the following is associated with cell death and NOT specifically with reversible cell injury?
a. [ ] Membrane blebs
b. [ ] Clumping of chromatin
c. [ ] Swelling of ER
d. [ ] Swelling of mitochondria
e. [X] Myelin figures
368. Which of the following is NOT associated with cell death?
a. [ ] Nuclear condensation (pyknosis)
b. [ ] Nuclear fragmentation (karyorrhexis)
c. [ ] Dissolution of the nucleus (karyolysis)
d. [X] Decrease in intracellular Ca++
e. [ ] Amorphous mitochondrial densities

45
Morphopathology - CS
Tamir Bessisy 1246

369. Which of the following causes of cell injury involves a single amino acid substitution in hemoglobin
S in sickle cell anemia?
a. [ ] Oxygen deprivation
b. [ ] Physical agents
c. [ ] Infectious agents
d. [ ] Immunologic reactions
e. [X] Genetic derangements
370. Which of the following would NOT cause mitochondrial damage?
a. [ ] Increase in cytosolic Ca++
b. [ ] Oxidative stress
c. [X] Retention of cytochrome c
d. [ ] Breakdown of phospholipids through the phospholipase A2 and sphingomyelin pathways
e. [ ] Lipid breakdown products (e.g. free fatty acids and ceramide)
371. Which of the following has a cheesy, yellow-white appearance at the area of necrosis and is
encountered most often in foci of tuberculous infection?
a. [ ] Coagulative necrosis
b. [ ] Liquefactive necrosis
c. [X] Causeous necrosis
d. [ ] Fat necrosis
e. [ ] Gangrenous necrosis
372. Which of the following is characteristic of focal bacterial or, occasionally, fungal infections, because
microbes stimulate the accumulation of inflammatory cells?
a. [ ] Coagulative necrosis
b. [X] Liquefactive necrosis
c. [ ] Causeous necrosis
d. [ ] Fat necrosis
e. [ ] Gangrenous necrosis
373. Chemically induced cell injury from carbon tetrachloride (CCl4) and acetaminophen (Tylenol)
affect which organ?
a. [ ] Brain
b. [ ] Kidneys
c. [ ] Pancreas
d. [ ] Spleen
e. [X] Liver
374. Which of the following diseases would most likely show glycogen abnormalities if tissues, such as
the descending loop of Henle and liver cells, are stained with periodic acid shiff (PAS)?
a. [ ] Chronic hypertension
b. [ ] Congestive heart failure
c. [ ] Abdominal aortic aneurysm
d. [ ] Rheumatoid arthritis
e. [X] Diabetes mellitus
375. Hemosiderin is a hemoglobin-derived, golden yellow-to-brown, granular or crystalline pigment that
can indicate a local excess of which of the following?
a. [ ] Oxygen
b. [ ] CO2
c. [X] Iron
d. [ ] Macrophages
e. [ ] Ca++

46
Morphopathology - CS
Tamir Bessisy 1246

376. Bilirubin is hemoglobin-derived and the normal major pigment found in bile. If found in excess,
what color does it change the skin?
a. [ ] Black
b. [ ] White
c. [ ] Red
d. [X] Yellow
e. [ ] Blue
377. A 48-year-old male with a history of chronic alcoholism who is continuing to perform adequately
on the job will most often have which of the following findings in liver:
a. [ ] Cholestasis
b. [X] Fatty change
c. [ ] Hemochromatosis
d. [ ] Hypertrophy of smooth endoplasmic reticulum
e. [ ] Coagulative necrosis
378. A 53-year-old male who is developing an acute myocardial infarction from coronary occlusion
has an irreversible injury to the myocardium when:
a. [ ] Glycogen is depleted
b. [ ] Cytoplasmic sodium increases
c. [X] Nuclei undergo karyorrhexis
d. [ ] Intracellular pH diminishes
e. [ ] Blebs form on cell membranes
379. After years of breathing dirty city air, your lungs have accumulated:
a. [X] Anthracotic pigment
b. [ ] Lipofuscin
c. [ ] Melanin
d. [ ] Hemosiderin
e. [ ] Biliverdin
380. The presence of squamous epithelium in the lower trachea of a 42-year-old female with a history of
smoking is called:
a. [ ] Dysplasia
b. [ ] Aplasia
c. [ ] Anaplasia
d. [ ] Hyperplasia
e. [X] Metaplasia
381. A 59-year-old female had a cerebral infarction. Months later, a computed tomographic (CT) scan shows a
cystic area in her cerebral cortex. The CT finding is a lesion that is the consequence of resolution from:
a. [X] Liquefactive necrosis
b. [ ] Atrophy
c. [ ] Coagulative necrosis
d. [ ] Caseous necrosis
e. [ ] Apoptosis
382. The light brown perinuclear pigment seen on H&E staining of the cardiac muscle fibers in the
heart of an 80 year old male is:
a. [ ] Hemosiderin resulting from iron overload
b. [X] Lipochrome from "wear and tear"
c. [ ] Glycogen resulting from a storage disease
d. [ ] Cholesterol as a consequence of atherosclerosis
e. [ ] Calcium deposition following necrosis

47
Morphopathology - CS
Tamir Bessisy 1246

383. Karyorrhexis refers to:


a. [ ] Disintegration of the cell cytoplasm
b. [ ] Cell membrane lysis
c. [X] Disintegration of the cell nucleus
d. [ ] Mitochondrial swelling and lysis
e. [ ] Oxygen toxicity
384. The spleen at autopsy on sectioning shows a tan to white, wedge-shaped lesion with base on the
capsule. This most likely represents the result of:
a. [X] Coagulative necrosis
b. [ ] Abscess formation
c. [ ] Metaplasia
d. [ ] Caseous necrosis
e. [ ] Liquefactive necrosis
385. A 3500 gm liver from a 35-year-old female has a yellow, greasy cut surface. This appearance most
likely resulted from:
a. [ ] Galactosemia
b. [ ] Iron accumulation
c. [ ] Mycobacterium tuberculosis infection
d. [X] Alcoholism
e. [ ] Hypoxia
386. A 73-year-old male suffers a "stroke" with loss of blood supply to cerebral cortex in the
distribution of the middle cerebral artery. The most likely consequence of this is:
a. [X] Infarction with liquefactive necrosis
b. [ ] Pale infarction with coagulative necrosis
c. [ ] Predominant loss of glial cells
d. [ ] Recovery of damaged neurons if the vascular supply is reestablished
e. [ ] Wet gangrene with secondary bacterial infection
387. Physical examination on a 42 year old female reveals scleral icterus. Which of the following
underlying conditions is most likely to contribute to this finding:
a. [ ] Hypercholesterolemia
b. [ ] Thrombocytopenia
c. [ ] Metastatic carcinoma
d. [X] Hepatitis
e. [ ] Diabetes mellitus
388. The appearance of fat necrosis is most often seen in which of the following settings:
a. [X] A 31 year old male has an acute abdomen with marked abdominal pain and an elevated serum amylase
b. [ ] A 66 year old female with chronic alcoholism has an elevated serum AST
c. [ ] A 23 year old female with a decreased total serum complement has a history of systemic lupus erythematosus
d. [ ] A 70 year old female with adenocarcinoma of the colon and metastases to liver has an elevated LDH
e. [ ] A 49 year old male with sudden onset of chest pain has an elevated serum creatine kinase
389. Dry gangrene is characterized by:
a. [X] Mummification
b. [ ] Proliferation
c. [ ] Hydration
d. [ ] Encephalomalacia
e. [ ] Myomalacia

48
Morphopathology - CS
Tamir Bessisy 1246

390. Fatty liver is characterized by:


a. [ ] Decreased liver size
b. [ ] Dense consistency
c. [ ] Rough surface
d. [X] Lipids in hepatocytes
e. [ ] Absence of nuclei
391. Myocardial fatty degeneration can be detected by the following stain:
a. [ ] Hematoxilin-Eosin
b. [ ] Picrofuchsin
c. [X] Sudan-3
d. [ ] Toluidine blue
e. [ ] Kongo-red
392. Clinical evidence of parenchymal lipidic degeneration of myocardium is:
a. [ ] Increased contractility
b. [ ] Hypertention
c. [X] Decreased contractility
d. [ ] Rupture of heart
e. [ ] Hyperemia
393. Liver steatosis is caused by:
a. [X] Alcoholism
b. [ ] Viral hepatitis B
c. [ ] Hypertention
d. [ ] Viral hepatitis A
e. [ ] All of the above
394. Parenchymal myocardial degeneration develops in the following situation:
a. [ ] Hypertension
b. [ ] Avitaminosis
c. [X] Diphtheria
d. [ ] Diabetes mellitus
e. [ ] Protein starvation
395. Liver steatosis is usually is followed by:
a. [X] Recovering of structures
b. [ ] Massive necrosis
c. [ ] Transformation in protein degeneration
d. [ ] Transformation into liver cirrhosis
e. [ ] Falls lobule appearance
396. Accumulation of lipids in the wall of the large arteries is typical:
a. [ ] Inflammation
b. [ ] Cachexia
c. [ ] Aneurism
d. [ ] Obesity
e. [X] Atherosclerosis
397. Which of the following processes is reversible?
a. [ ] Apoptosis
b. [X] Mucoid intumescence
c. [ ] Hyalinosis
d. [ ] Amyloidosis
e. [ ] Fibrinoid changes

49
Morphopathology - CS
Tamir Bessisy 1246

398. Heart valves hyalinosis is typical:


a. [ ] Congenital heart disease
b. [X] Rheumatic fever
c. [ ] Alcoholism
d. [ ] Hypertensive disease
e. [ ] Cardiomyopathy
399. Systemic arteriolar hyalinosis is typical:
a. [ ] Atherosclerosis
b. [ ] Tuberculosis
c. [ ] Alcoholism
d. [ ] Syphilis
e. [X] Hypertensive disease
400. The following structures are subject to hyaline changes:
a. [ ] Renal stones
b. [ ] Bone tissue
c. [ ] Amyloid
d. [ ] Cartilaginous tissue
e. [X] Fibrous tissue
401. Amyloid is a protein that deposits in the following structure:
a. [ ] Cells
b. [ ] Foci of necrosis
c. [ ] Nuclei of cells
d. [ ] Foci of calcification
e. [X] Interstitial tissue
402. Which of the following is amyloid specific stain?
a. [ ] Hematoxylin-eosin
b. [ ] Picrofuchsin
c. [X] Kongo-red
d. [ ] Toluidin
e. [ ] Sudan-3
403. Amyloidosis is a complication of the following disorder:
a. [ ] Pneumonia
b. [ ] Hypertensive disease
c. [ ] Dysentery
d. [ ] Aterosclerosis
e. [X] Bronchiectasis
404. Secondary amyloidosis can be a complication of:
a. [X] Tuberculosis
b. [ ] Atherosclerosis
c. [ ] Diabetes mellitus
d. [ ] Hepatitis
e. [ ] Hypertension
405. Generalized obesity contributes to:
a. [ ] Brown atrophy of the heart
b. [ ] Acute pancreatitis
c. [ ] Myocarditis
d. [ ] Goiter
e. [X] Ischemic heart disease

50
Morphopathology - CS
Tamir Bessisy 1246

406. Pigments are substances that:


a. [X] Are collared
b. [ ] May accumulate color
c. [ ] Of a proteic nature
d. [ ] Care dissolved in lipids
e. [ ] All of the above
407. Brown induration of lungs is characterized by accumulation of:
a. [ ] Hydrochloric hematin
b. [ ] Lipofuscin
c. [ ] Bilirubin
d. [ ] Coal dust
e. [X] Hemosiderin
408. Which pigment appears in the area of hemorrhage?
a. [ ] Adrenochrom
b. [X] Hemosiderin
c. [ ] Melanin
d. [ ] Lipofuscin
e. [ ] Lypochrom
409. Dystrophic calcification is referred to:
a. [ ] Accumulation of calcium salts into unmodified gastric mucosa
b. [ ] Calcareous metastases in the kidneys
c. [X] Necrosis foci calcification
d. [ ] Accumulation of calcium salts into unmodified lungs
e. [ ] Accumulation of calcium salts into myocardium in condition of hypercalcemia
410. Necrosis etiology:
a. [ ] Biologic factors
b. [ ] Blood flow disturbances
c. [ ] Allergic factors
d. [ ] Chemicals
e. [X] All of the above
411. Humid gangrene is found into:
a. [X] Intestine
b. [ ] Brain
c. [ ] Kidney
d. [ ] Myocardium
e. [ ] Liver
412. Which of the following disorders is manifested by humid necrosis?
a. [ ] Tuberculosis of lung
b. [ ] Rheumatic pericarditis
c. [ ] Myocardial infarction
d. [ ] Spleen infarction
e. [X] Ischemic infarction of brain
413. Which of the following disorders is manifested by caseous necrosis?
a. [X] Milliary tuberculosis of lung
b. [ ] Myocardial infarction
c. [ ] Dysentery
d. [ ] Typhoid fever
e. [ ] Gangrene

51
Morphopathology - CS
Tamir Bessisy 1246

414. Unfavorable outcomes of necrosis include:


a. [ ] Organization
b. [ ] Calcification
c. [X] Purulent fusion
d. [ ] Bordering inflammation
e. [ ] Sepsis
415. Localization of gangrene:
a. [ ] Kidney
b. [ ] Myocardium
c. [X] Soft tissues of the lower extremities
d. [ ] Brain
e. [ ] All of the above
416. A patient with a decreased number of red blood cells would be most likely to have problems with
which of the following?
a. [ ] Antibody production
b. [X] Oxygen delivery to tissues
c. [ ] Clot formation
d. [ ] Bacterial infections
e. [ ] All of the above
417. The precursor of all lines of blood cells is the _______________.
a. [ ] Myeloblast
b. [X] Hemocytoblast
c. [ ] Proerythroblast
d. [ ] Progranulocyte
e. [ ] Lymphoblast
418. When red blood cells are worn out, part of their components are recycled while others are disposed.
Select the INCORRECT statement about destruction of red blood cells.
a. [X] The greenish pigment, biliverdin, is recycled to the bone marrow.
b. [ ] Iron is carried to the bone marrow by a protein called transferrin.
c. [ ] Biliverdin and bilirubin impart color to bile.
d. [ ] Macrophages in the liver and spleen destroy worn out red blood cells.
e. [ ] All of the above
419. Which dietary component(s) is/are needed for DNA synthesis, and thus greatly influence the
production of red blood cells?
a. [ ] calcium
b. [ ] iron
c. [X] vitamin B12 and folic acid
d. [ ] protein
e. [ ] lipids
420. The type of anemia that is fairly common and caused by insufficient dietary iron is:
a. [ ] Aplastic anemia
b. [ ] Pernicious anemia
c. [ ] Hemolytic anemia
d. [X] Iron deficiency anemia
e. [ ] All of the above
421. The type of white blood cell that often arrives at the site of infection first, is a granulocyte, and
contains granules that stain light purple is a:
a. [ ] Basophil
b. [ ] Eosinophil
c. [X] Neutrophil

52
Morphopathology - CS
Tamir Bessisy 1246

d. [ ] Monocyte
e. [ ] Histiocyte
422. The largest cells in the blood that leave the bloodstream to become macrophages are the:
a. [ ] eosinophils
b. [ ] monocytes
c. [X] basophils
d. [ ] neutrophils
e. [ ] epitheliocyte
423. A person with eosinophilia, or greater than normal numbers of eosinophils, is most likely suffering from:
a. [X] Allergies or internal parasites
b. [ ] Anemia
c. [ ] An autoimmune disease
d. [ ] Diabetes
e. [ ] Syphilis
424. Which blood cell can be described as being a biconcave disc?
a. [ ] Platelet
b. [ ] Neutrophil
c. [ ] Eosinophil
d. [X] Erythrocyte
e. [ ] Macrophage
425. What is the term for erythrocytes of various sizes?
a. [X] Anisocytosis
b. [ ] Poikilocytosis
c. [ ] Anemias
d. [ ] Polycythemias
e. [ ] Thalassemia
426. Large, abnormally shaped erythrocytes, normal hemoglobin concentrations?
a. [ ] Normocytic-normochromic anemia
b. [ ] Microcytic-hypochromic anemia:
c. [X] Macrocytic normochromic anemia
d. [ ] Macrocytic hypochromic anemia
e. [ ] Microcytic hyperchromic anemia
427. Why does B12 and folate deficiencies cause anemia?
a. [ ] Stem cells are unable to differentiate into erythrocytes
b. [ ] Rbc's have malfromed hg molecules
c. [X] Because erythrocytes have a shorter life span and die prematurely, this decreases their number
in the circulation causing anemia
d. [ ] Rbs have decreased o2 carrying capacity
e. [ ] Rbs have increased o2 carrying capacity
428. Which of the following diseases may be associated with anemia
a. [ ] Chronic renal failure
b. [ ] Hepatic failure
c. [ ] Gastric cancer
d. [ ] Chronic leukemia
e. [X] All of the above

53
Morphopathology - CS
Tamir Bessisy 1246

429. Which type of leukemia is characterized by a combination of: a phasic clinical course, blast crisis, pus
like bone marrow, a sharp increase in weight of the spleen and liver, diffuse leukemic infiltration in the
liver along sinusoids?
a. [ ] Chronic lymphocytic leukemia
b. [ ] Acute eritromieloblastic leukemia
c. [ ] Acute lymphoblastic leukemia
d. [X] Chronic myelogenous leukemia
e. [ ] Chronic erythroleukemia
430. All of the following statements are true regarding lymphoma EXCEPT:
a. [ ] Hodgkin's lymphoma generally has a good prognosis
b. [ ] The malignant cell of Hodgkin's lymphoma is the Reed-Sternberg cell
c. [X] Low grade lymphomas frequently can be cured by chemotherapy
d. [ ] Follicular lymphomas are all B-cell lymphomas
e. [ ] The malignant cell of Hodgkin's lymphoma is the Hodgkin cell
431. Metaplasia is:
a. [X] The replacement of one differentiated cell type with another
b. [ ] The transformation of a cell type to malignancy
c. [ ] An irreversible cellular adaptation
d. [ ] The disorganization of cells into various sizes, shape and arrangement
e. [ ] Degenerative cell derangements
432. Platelet-derived growth factor is best-known for
a. [X] Activating fibroblasts in wound healing
b. [ ] Activating the complement cascade
c. [ ] Aiding the maturation of granulomas
d. [ ] Causing epithelium to heal over breaks
e. [ ] Producing hemostasis in minor injuries
433. Which of the following is NOT capable of regeneration?
a. [ ] Epithelial tissue
b. [X] Cardiac tissue
c. [ ] Skin
d. [ ] Liver
e. [ ] Kidney
434. Which of the following is the most true regarding scar formation (net collagen accumulation)?
a. [ ] Depends on increased collagen synthesis only
b. [ ] Depends on decreased degradation only
c. [ ] Depends on either increased collagen synthesis or decreased degradation
d. [X] Depends on both increased collagen synthesis and decreased degradation
e. [ ] Depends on neither increased collagen synthesis nor decreased degradation
435. Which of the following is true when comparing healing by second intention (separated edges) to
healing by first intention (opposed edges)?
a. [X] Significant granulation and wound contraction are seen in first intention
b. [ ] A scab forms within 24 hours in first intention
c. [ ] A fibrous union is seen in first intention, weeks after injury
d. [ ] In first intention, mitosis near the wound is seen in 3-7 days
e. [ ] In first intention, new capillaries form in 3-7 days
436. Which of the following contributes to wound tensile strength toward the end of healing?
a. [ ] Increased collagen synthesis
b. [ ] Decreased collagen degradation
c. [X] Structural collagen fiber modification
d. [ ] Decreased collagen cross-linking

54
Morphopathology - CS
Tamir Bessisy 1246

e. [ ] Decreased collagen fiber size


437. Which of the following differentiates fibrosis from normal wound healing?
a. [ ] Cutaneous scar-cell proliferation
b. [ ] Cell-cell interactions
c. [ ] Cell-matrix interactions
d. [X] ECM deposition
e. [ ] Immune and autoimmune reactions
438. Physiological hypertrophy of the myocardium is caused by:
a. [ ] Heart vices
b. [ ] Cardiosclerosis
c. [X] High physical activity
d. [ ] Hypertension
e. [ ] Toxic myocarditis
439. Specify the levels that regenerative recovery can occur:
a. [ ] Molecular
b. [ ] Cellular
c. [ ] Tissue
d. [ ] Organ
e. [X] All of the above
440. Decompensated heart develops:
a. [ ] Increasing the monocytes number
b. [ ] Atrophy of myocytes
c. [X] Increased size of myocytes
d. [ ] Degeneration of myocytes
e. [ ] Inflammation of myocytes
441. Neurohormonal hypertrophy develops in the following organs:
a. [ ] Heart due to hypertensive disease
b. [X] Mammary glands in pregnancy
c. [ ] Urinary bladder in prostatic hypertrophy
d. [ ] Remained kidney
e. [ ] Wall of the stomach due pyloric stenosis
442. Reduction of functioning structures volume is called:
a. [ ] Hypertrophy
b. [ ] Hypoplasia
c. [ ] Hyperplasia
d. [ ] Dysplasia
e. [X] Atrophy
443. The generalized atrophy causes include:
a. [X] Nutritional exhaustion
b. [ ] Atrophy due to inactivity
c. [ ] Neurotic atrophy
d. [ ] All of the above
e. [ ] Atrophy due compression
444. Atrophy due to compression develops in the following case:
a. [ ] Radiation sickness of bone marrow
b. [ ] Muscle due to fracture
c. [X] Kidney due to stones
d. [ ] Myocardium due to atherosclerosis

55
Morphopathology - CS
Tamir Bessisy 1246

445. An example of atrophy from lack of blood supply is:


a. [X] Focal atrophy of the myocardium due to coronary artery atherosclerosis
b. [ ] Atrophy of the adrenal cortex due to corticosteroids administration
c. [ ] Muscle atrophy of bone due tofracture
d. [ ] Atrophy of the optic nerve by removing eye
e. [ ] All of the above
446. Transition of one tissue type to another related is called:
a. [ ] Dysplasia
b. [X] Metaplasia
c. [ ] Anaplasia
d. [ ] Malignancy
e. [ ] Hyperplasia
447. Connective tissue metaplasia is available at:
a. [X] Bone
b. [ ] Epithelium
c. [ ] Muscle
d. [ ] Nerve
e. [ ] All of the above
448. Which of the followings may develop on the background of bronchial epithelium metaplasia?
a. [ ] Dystrophy
b. [X] Malignant neoplasm
c. [ ] Atrophy
d. [ ] Inflammation
e. [ ] Necrosis
449. Disturbance of cell proliferation and differentiation with the advent of cellular atypia in some cells is called:
a. [ ] Hyperplasia
b. [X] Dysplasia
c. [ ] Metaplasia
d. [ ] Organization
e. [ ] Anaplasia
450. Which of the following is wound healing type?
a. [ ] Due to organization
b. [X] Primary intention
c. [ ] Capsulation
d. [ ] Metaplasia
e. [ ] Al of the above
451. Restoration of structural elements instead the lost tissue is called:
a. [ ] Organization
b. [ ] Dysplasia
c. [X] Regeneration
d. [ ] Anaplasia
e. [ ] Metaplasia
452. What granulation tissue means?
a. [ ] Fibrous connective tissue
b. [X] Young connective tissue
c. [ ] Mature connective tissue
d. [ ] Poor vascular tissue
e. [ ] Poor cells tissue

56
Morphopathology - CS
Tamir Bessisy 1246

453. What can tissue type regenerate by restitution?


a. [ ] Nervous
b. [ ] Cartilage
c. [ ] Muscular
d. [ ] Myocardium
e. [X] Blood
454. Duration of blastogenesis is:
a. [X] Two weeks
b. [ ] three weeks
c. [ ] four weeks
d. [ ] five weeks
e. [ ] Six weeks
455. Complete absence of an organ is:
a. [X] Agenesis
b. [ ] Atresia
c. [ ] Dysplasia
d. [ ] Hyperplasia
e. [ ] Metaplasia
456. Hypoplasia is:
a. [ ] Complete absence of an organ
b. [ ] Absence of an opening
c. [X] Incomplete development or under- development of an organ with decreased numbers of cells
d. [ ] Overdevelopment of an organ associated with increased numbers of cells
e. [ ] None of the above
457. The following signs are related to prematurity:
a. [ ] A little mass and length of child (less than 2,5 kg and 47 centimeters)
b. [ ] Superfluous fluff is saved, especially on a shoulder girdle and superior portion of the back
c. [ ] Absence or weak expressed of ossification nuclei
d. [ ] Nose and ears cartilages mildness
e. [X] All of the above
458. Which of the following statements is true about seminoma
a. [ ] It is a benign tumor of the seminiferous tubules
b. [ ] It is a radioresistant tumor
c. [ ] It is most commonly affects adults below the age 30 years
d. [ ] All of the above
e. [X] None of the above
459. Which of the following statements is true about Benign Prostatic Hypertrophy
a. [ ] It predominantly affects the peripheral prostatic zone
b. [ ] It very commonly changes into prostatic carcinoma
c. [X] Untreated cases may be complicated by renal failure
d. [ ] All of the above
e. [ ] None of the above
460. Which of the following statement is true about prostatic carcinoma?
a. [ ] Most cases are estrogen dependent tumor
b. [ ] Most cases arise in peripheral prostatic zone
c. [ ] Gleason combined grade 3+1 (total 4) means high grade tumor
d. [X] All of above
e. [ ] None of above

57
Morphopathology - CS
Tamir Bessisy 1246

461. The macroscopic pattern of prostatic hyperplasia includes the following EXCEPT:
a. [ ] Increased in size
b. [ ] Nodular
c. [ ] Dense consistence
d. [X] Brown colored
e. [ ] Contains fibrous tissue filaments
462. The complications in Prostatic hyperplasia are the following EXCEPT:
a. [ ] Urine retention
b. [X] Urinary bladder wall atrophy
c. [ ] Urinary bladder wall hypertrophy
d. [ ] Pyelonephritis
e. [ ] Cystitis
463. Which aortic segment is most common involved in Syphilis?
a. [X] Ascent
b. [ ] Descendent
c. [ ] Thoracic
d. [ ] Abdominal
e. [ ] Femoral
464. Which type of pneumonia can develop in congenital syphilis?
a. [ ] Lobar
b. [ ] Interstitial
c. [ ] Necrotic
d. [X] Alba
e. [ ] Bronchopneumonia
465. Sepsis types according to entry of infection:
a. [ ] Therapeutic
b. [ ] Surgical
c. [ ] Gynecologic
d. [ ] Umbilical
e. [X] All of the above
466.What inflammation type underlies anthrax carbuncle?
a. [ ] Serous
b. [ ] Purulent
c. [X] Hemorrhagic
d. [ ] Putrid
e. [ ] Catarrhal
466. In septic (bacterial) endocarditis is observed:
a. [ ] Acute verrucous endocarditis
b. [ ] Chronic verucous endocarditis
c. [X] Polypous ulcerative endocarditis
d. [ ] Ethyl fibroplastic parietal endocarditis with eosinophilia
e. [ ] Diffuse endocarditis
468. Which of the following organisms is the MOST COMMON cause of nosocomial bloodstream infections?
a. [X] Staphylococcus epidermidis
b. [ ] Escherichia coli
c. [ ] Enterococcus faecalis
d. [ ] Clostridium perfringens
e. [ ] Peptostreptococcus

58
Morphopathology - CS
Tamir Bessisy 1246

469. Which of the following are TRUE of sepsis?


a. [ ] Disseminated intravascular coagulation (DIC) exhibits widespread clotting in capillaries.
b. [ ] Disseminated intravascular coagulation (DIC) results in hemorrhage.
c. [ ] Clinical trials of patients with severe septis have shown positive results with heparin
treatment to treat microembolisms.
d. [X] A and B
e. [ ] All of the above

59

Vous aimerez peut-être aussi